Вы находитесь на странице: 1из 297
ol GILLESANIA. ey Civil . Sau s§ Reference ere ety tery to Past CE Board SCT ea q ERT AAT | MEUM seeer i UCT} Pe een] Geotechnical Engineering Structural Engineering and Construction Civil Engineering Reference For Leense Exaninaons Volume 2 Copyright © 2006 by Diego Inocencio Tapang Gillesaaia All righis reserved, No part of this book aay be ISBN 971-8614-34-6 (Cover design by the auhor. Table of Contents CE Board. May 2000... Mathers Srvying and Teanspe 1 to 33 Structural Enpnecringand Consrction Solution. CE Board November 2000... Mathematia Surveying nd Teaneporacon Enghnetscg, jan. 34 to 62 Structural Fig snot a Solutton: CE Board May 2001... 63 to 97 Mathernatics, Surveying, and Teapot Engine 83 Solution... Ean Hydraulics and Geotechnical Engineering. ie 76 Solucion si 80 Seruetural Engincering and Construction ee Br Solution ee at eee een CE Board November 2001... ... 98 to 137 Mathematics, Surveying, and. Tranporton Engen 98 Solution... ees “Hydraulics and Geotechnical Engineer Solution ‘Structural Baie and Construction... SowtIOM ens = im Table of Comtents CE Board May 2002..... ..139 to 187 Mathematics, Surveying sel Teapraton Engoeeig 339 aes fanical En, i Hydraulics and Geotechnic cing. eres ons 160 Structural Engineering and Construction. _ 169 ‘Solution. i vs vt mien mom LEE CE Board November 2002... «188 to 237 Mothematis, Suen. and Transporation Race lB ‘Solution. eine 192 ‘Hydraulics and Geotechnical Engineering. eer ‘Solution. 208 ‘Structural Eaguecring tl ‘Conseruction. seinen serene ee LAD, Solution...... - i229 CE Board May 2003... Mathematics, Sureying and Transportation Engine ‘Solution... Hydraulics and: ‘Geotechnical Engineering, Solution, = . 258 ‘Structural Engineering and Construct 269 Solution. 280. CE Board November 2003 292 to 339 Markemaic,Sureing an Transpo deen Solution. ~ = es Hydraulics and Geetc es 304 Solution... _ <3 Stniceural Englacering snd Construction ees oe 32 Seite ats ee BL m 341 to 388 341 SE Sa Structural Engincering and Construction Solution... 33 381 CE Board November 2004... 389 to Be athena, Sumejing and Transporasies 369 394 403 z ero: Stmnual Engine ard Contraction 1. = 420 Solution... - 426 CE Board May 2005... ere to ney Mathematics, Surveying, and Transpo ar Solutio 482 “Hydraulics 453 Solution... nd Struccural Engineering and Construction 0... 466 SSOREIOR rn : = 474 CE Board November 2005...... 481 to 526 Matmatis Surveying and Transporation Engineing.. 481 Solution, a as, ‘Hydraulics and Geotechnical Engineering = vn B Solution . a 501 310 Structural Engineering and Construction Solution, = 5? Problem Index. civil 1 Vol. 2 Seat Now CIVIL ENGINEER Licensure Examination ‘Thursday, May 11, 2000 06:00 a.m. - 01:00 pm. MATHEMATICS, SURVEYING & TRANSPORTATION ENG’, SETA CTION: Sel lect the correct answer for cach of the following questions, for each item by’ shading the box corresponding to the letter of your choice on the answer sheet provided. STRICTLY NO FRASURES ALLOWED, Use pencil no, 2only. MULTIPLE CHOICE 1. How long will it take for money to quadruple itself if invested at 20% pone aan? A. 107 years 2 C 95years B, 62 years D. 7.1 years“ 2. Find the area in sq. m, of a spherical triangle of whose angles are 123°, 64°, and 73", The radius of the sphere is 301m. 4. 18633 ©, 19586 B 15708 B. 1802 3. Conwert 405" to mils ral i D, 200 mls es wee of a ight circular cone of radius 4 cm i on slant ght * C san D. idem 5. Five thousand pesos is deposited at the end of each year for 15 years into an. ‘account eaming 75% compounded continuously. Find the amount after 15 ‘years. A, P1393 5413 © r18.265.9 B. Pisz7542 D. P1854 6 There are four geometric means between 3 and 729. Find the fourth term, A. Bl C23 Pizt ons a) 7, Loge9?5=x, Find x Ras © 57 Bana D. 287 a 5 Evaluate [52 cose a A 15401 © cos B. 19086 DB. 2412 9 projectiles ire tan angle of 20 with she horizontal atthe top of 30m high bulding, ‘The muzzle velocity is 500 m/s. What the foal time of ne Z os © 2128 a B B75 Di. 1855 io * z May 2000 10. Tore vertices of a triangle are (6/-1) and third vertex such that the centroid of the triangle 13 C13 480 000 has aie expectancy te rt cost What ic te book the decinig blance method? ©. Fisesaz B Pisa 758 D, F2a.785 ‘Twa sides of a tangle measure 18 cm and cm. The third side may be: AL 12, 10 BB Bw At what walue of x will the slope of the curve 2° =9x~ y= Obe 18? re C5 Bis Ba |. The frustum of a regular triangular pyramid has equilateral bsiangles for its bbases and has an altitude of 6 m. The lower base edge is 9m. Ifthe volume is 288 cum, what isthe upper base edge? incler of radius 6-m hes its axis along, the X-axis. aA second o sn rode has its axis along the Y-axis, Find the volume, in the frst tecommen to the two cylinders, : PA ©. 14m A Bu D. 28m 1h A lighthouse js 2m off a straight shore search lig at the lighthouse fecuber 10 « long the shore. When the car is 1 km from the point nearest tothe lighBouse, the searchlight rotates 0.25 rev/hour. Find the speed of ‘A358 © 22 B 255 D. 337 The total surface area of a closed cylindrical tank is 153.94 square meter, IE the volume is to be maximum, what is its height in meters? A 68m, +B. 57m ‘Determine the moment of inertinjabout the X-axis, of the area bounded by the curve x= 4y, the line x= 4, and the X-axis, 985 © 1017 B, 13.24 D219 Civil Engineering 3 Reference Vol. 2 2 From point A ona simple carve, the perpendicular distance to the fa a is Gt mt The tangent passes though the PC The distance from rs am, Find the length ofthe curve from PCto A, inm. © 25855 D212 B 35247 The ground makes a uniform sly 240. "AUSTA 12+ 180, the center the roadway is 12m fill, Avthe other station, the center height is 25 m cut. Find the length of cut in meters, A. 30.85 ©, 4632 B 40.54 B. 5028 Avert parbolosag carve hat tangent grades of 12% and 406%, Ihe changes uniformly at 0185 per 20m fin the length of Ue curve, ners re eam 3 150m D. 20m ae observed mri anges of « angular pie of ld ABC are as 7 C= as" 15°05" 8 os ‘The most probable value of angle Bis nearest 96° 30° 06" 1D. 96° 30/12" Acclosed traverse has the following data: Line Bearing aR BC [ay ra E ‘ofan ellipse is 28.448 unit the minor axis? Ds of the north expresseay 1200 vehicles hour. Is 25 vehicles per Komnster, what isthe space mean sped the die of ae e driver of car travelling at certain speed suddenly sees an obstruction headed atlas co 7 dg th prepona s ‘of 25 seconds, Determine the car’a speed of approach in kph. A. 856 C108 y B63, D. 1243 A certain copier machine cost P1S0.000 with a trade-in value of P15,000 after ‘aking 800,000 copies. Using, the declining balance method, what i the book-value when the machine hed made 300,000 copies? A. PSSAIL © e251 BL P64.896 Di, 63.254 ' | Mochematles, Surveying, 20d Civil Engineering Solutions to May 2000 Examination Te equnin of he dri theca = 14 D 1 cicula eylndar whose Literal area is 25918 m?* ime of arigh aa ‘of 7.068 0 ase | Let be the marmber of yas: D. is12m 7.1 years m2 7 B= A484 C—190" = 123+ 84° £73°- 190°= 100" (30)*(100") 180° Areas =1570.8.0q. m. ae log 1975 =r 37 log 975 / log 8= 3.31 e712 @ ‘Thus, among the choices, x may be 13 8 May 2000 ‘Transportation Engineerios me Si tava, aval ae Javdae aa dO/ dé = 0,25 rev/ hour 0 /dt= 052 rad hoor ee ee has: tan @=x/2= 05x | x Bost en no 2 055 aie When x =1;0 = 26.565° se 26560" (052) = 05 ts ses 3:93 key # a = For a given total surfac the volume is a maximum when the ‘Hameter (D) equals the he i Ap= $DEN 24 aDH=2 4 HP+ a(HVH) = 153.98 Hes7m m9 yay «[-ayreety) 1238 ey ie af Civil Engineering: Reference Vol. 2 20 Insight triangle ABO: Res 2604 y Re ~ 67600 RE~ 126R + 4056 R= 560.125 m sin 8 = 260/56025 Cera point is a a the grade diagram rome grade diag shown: L 20 Ta+06 T18 = 200m 10 May 2000 “Transportation Enginecring ¥ angles = 35° 14°37” + 96" 30" 09" +48° 15°05" E angles ~ 129.9975" = 179" 59° 51” ‘Since the sum of the interior angles of a triangle is 1807, the total error Eis, E= 180° - 179° 99 51” = 0° 0 09" 8 30" 09 + (0° 0 09")/3 = 96° 30°12" Most probable value of ‘Most probable val Solving for the closing line AC Tine Bearing Distance Latitude Departure CD SIPWE | 4S an7od2 19 DE S70 W 5845 18.7505 9 FA N7at30"W. 5000133619 48.1815 i NeW _ eee ‘Sum ~i8.1828 86.0702 eee Latirude of AC = +48.1828 Departure of AC= +88,0702 Bearing of AC= N61 Laemgth of AC = (asn626)"+(@80702)" = 100389"m In iangle ABC. 100.385 = 60! + 72.6% -2(60)(7269) cos Basra” 7269 __ 100.389 pa =05 897 Op =81c-0= 45,8292" = 154805" ac = B- Gan = 97.8683 ~ 15.4808 ~ 82.3875° Bearing ofling BC= S82 25°F Brose» ® 150,000(1-0,00000287823) 00 BVazasco™ PSS,2585 Balby 4a“ 16 ane Equation of diveetsix = y =4tory+4=0 13 in diameter is divided into tw 1 cha into wo ual chanbe hee, ane provided wih abn ihc iat tion Ata certain wel in one chamber 12 May 2000 Geotectinical Rogineering eo Bet os CIVIL ENGINEER Licensure Examination ‘Thursday, May 11,2000 €. 3565 vORAULICS & GEOTECHNICAL ENGINEER sera 2 C Bes INSTRUCTION: Select the coect anne for cach of he fll ing sions See eg haw ee a Vicon Lar ing be box corresponding tthe eter Oe seer he eerettoice on theanswer sheet provided. ttent rapes aston, By Tew much used ost SE RE URES ALLOWED. Use pencil 0,2 ly cones Would the dicharge 0 wt D. ost ma MULTIPLE CHOICE wo, eel! tion 6 m at the side of = tank contain age ot cevation dm reads 100 KPa. aa? 1A pressure gage at te AE LE “Anote Safe gram of te ig Sein eee ba 8 im De madeA0kre Wintec lon appesame/e ‘are as follows: Ly = 1500 mim, f= 0.025; Ls = 1a Dita 2 ood Moats in water wth 20-cm projecting abowe the water cat oa ot in ea wl len wa cm prejeting Above cit footing is to support two square columns each 12° « 12° and ‘What is the ight ofthe block? ers. One column © 50cm 5 Sam STegaly connected in horizontal 30-mame-damneter pipe, Pea 3 Waters legally Svs to gages ODO apart ahowed n diference of, [70 APs store erties from the const connection two gages 400m apast showed a difercnes es Speer ee pele Co orang f= OS, how wach water i Dein stoke frm he Besermine the length ofthe footing ifthe ‘A. 1834 6 ‘ond diameter are connected is how many times greater than i How long will t tke for the water i 20 min he spl wth cet elevation of 5 oe frou iotcese cee Ute teres one eee fon chon Paice formats, Ann Cmaentresctoxeurteccareal 125 Water flows over a spillway and into & Giergoes a jump, whatis the depth of flow ‘A. exitical c wes flood runoff from a , supercritical D. ee 6. a tiapefoidal canal svith sides sloping base width of 2m. the cee 53h eet is Tm, whats the hydraulic radius for this constion? Tae D. 3.28 eno yeni as 14, ‘The ratio of tke volume of water to the volume of voids ea , 086 m A, moisture content degree of saturation 7. -Amasonry dam 4m of top, 16 at the bottom and 24-m high bas eater 21 By void eat D. porosity Araagonry dam dae” nepecting hydrosiabc opi, what she 08 eo etna in eg? Assure lg of sony me 200 ce 4. 1530 < 32200 B. 34,750 1D, 39,780, 1 the following most nearly gives the maximum dry unit weight of in grams a. B16 16. Weter. flowy in a 10 feet wide recta persccond, What is the critical ds ‘A, LOaf te of 64 cubie feet Die sure content of 20% and degree of saturation of v7, 7 oe Spomteg of 261" Devermine the oy unit we me 934 (a D.1025 1a A tact of soi ly is found wo Imes mols conunt of 300% degree clay i Loon opeciic gravity of the solids is 276 Determine the voids ratio of this soil. 6063 D, 0472 pacted fil of sandy clay gives the following results: Weight of moist sil from the hele = 1088 grams ‘Oven-dnied weight of the grams Volume of test hole ~ 001 a maximum dry 5, Whieh of the D. 93% Civil Engineering eee eee ee a ee 2 harge over a rect ‘indy doc tthe Sree head cr Conder the vlaty of eproach and use the Astin 3 wee Winter flows trough an orifice atthe etc side of at toil head of 1. The jet trikes a pent 25. Stowe tne ortee tnd 25 izontally away from the vena contract. What isthe value of the head? ‘A. O85 m1 072m B 054m DL 0.63 m. END" 16 May 2000 Solutions to May 2000 Examination ma m2 Lat = appr of heb Inwater iar vo= 8 AGH = 20) = SAH) Hoes 9 0) Ino id (2080) 7e Ht H=dem as Upton fom te ale te BE 0 as427m : =1427; Qu=0216m'/s pon MESON ay yan Quins = Qu= Qo= 01216 - 0.205 ~ 0.011 mi¥/s iin ~ Hi liters per second. ‘Civil Engineering si Reference Vol. 2 7 Ga a? bfi= is 10.0826 f;L12)7 _ 0.0826 fz130;7 OM = 1543,500 kgm kyz = RM-OM $576,000 = 5,990,400 -1/543,500; = = 772m e-B/2-3 ~028m Civil Engineering 19 Reference Vol. 2 a0 Wfy= 624 m 0.0826(002)1500)0;? _ 55 4 oe Qi= 1390/5 fem 256 = 20825(0.025) 1000907 fim rag m HOBBES I000}5" (0.45) — /2Ke/2) 5-0) aan as 0.11 6 te 59 see To solve the eleoation of reservoir €; fp QORZAOOIEY OVO 55 5 ms L Sap ou nr gealmasin waxed sneer fsa Q- =KARY 0,000. |, For th taper sion: te 732) Locate the resultant fad by taking moment about the 40-kip load: 90 x= 40(0)+ 50(12); x= 6.67 feet From the figure: L/2=25+667; L= 18.34 feet Increase in flow = 0.61 m¥/s : To compl of footing Another sohution: in | (62-4 x 2.4)(2)~ (62.4% 1.78) ~2) = 1367-266 pal Using the relationship, 5 Total load = 40 + 50 90 kips = 90,000 Ibs 90,000 Qine o4 2714; Ong = BBLAD MLE . Atwent * 357 064 poe 3 LxW= 1834 x W= 6582 W359 fect yarauiles ana ak Grotechaleal Engineering Riemer ieee 5 _ 085-0613 mig O8S=040 : aaa ee 0527 iB | B 20 as Sed a Tr tr _ xs0000)f 11) 1 seam wae aL lz | 250) ~ 478.55.cc feo= 11938 pet a * 100% = 99.3% an First we neglect the velocity ofa Er peck ty OF approach to get the approsimate velocity 053= 184 L[(03 + ooosi86)y"— 5 ca16 lees seule eee 86}? (0.003186)/] = 64/10 = 6410/3 per foot wi | seo 640011 Pe ae adhe «fe ~ rss Pot he vnter yg Vs | a m7 GMC Sq. 201H3}=0066 «174 ane dic ca “Seca m1 | GMC~Se 2764) ORSe; = 1299 22 May 2000 ‘sna Constewetion ‘Seat Noe CIVIL ENGINEER Licensure Examination Sunday, May 12,2000 1. SBERIN Ui SETA correct answer for each of the fallowing, questions. ‘Mark galy-one answer for each item by shading the box corresponding, to the leiter ot yo toe onthe brceret sheet proved STRICTLY NOERASURES ALLOWED. Use pencil no. 2 only. MULTIPLE CHOICE Situation I In the connection shown in Figure ST-23, # load transmits 200 KN, of 200 mm, For his problem 2B = 20 ma ‘The loed és transmitted to the column by the plates and §-22 mm diameter rivets. The plates are adequate to transanit the load to the rivets. The ‘Connection can be analyzed by replacing the given load wath an equiva ‘Hical force alone acting through the centreid of the ig mort ay gives the mass sna sss the rivets in MejaPascals for equivalent vertical force alone acting on the ‘centroid. B iso D.70. 2. Whi of the felewing mast nary gives he raadmur shear stress in the rivets in MegaPascals for the equivalent moment alone. ‘A 700, C720 B74 D. 760 3. Which of the following most nearly gives the maximum shear stress in the rivets in MegaPrascals ‘A170 10 B 160 B. 130 Situation 2 - The system shown in Figure ME-Q8 consist of rigll bars AB, AC BD, CO hinged at points A, & © and D. When Ps zero, points # aod C arecoincident (ue. A.B, C, & D lies ona stiaight line) and the tension on AB is 200/N. For the given arrangement in the figore: 4. Which of the following most nearly gives the tension on member 48 in N. A. 80 C900 B. 1130 Di. 1020 '5. Which of the following most nearly gives the value of Pin N. A B40 C. 860 B. 1020 D. 1540 & Which of the following most nearly gives the spring constant X for system ABCD in.N/m. ‘M50 c 200 1870 B, 2560, 7 Civil Engineering Reterenss VoL 2 23 ‘Situation 3 The three concurrent forces shown in he figure are in equilibrium, 7 Which ofthe follwing most manly pivesthevalue srangie. Coe Di ae” following most nearly gives the valueof angle C7" ©. 76° 9. Which of the following most nearly gives the vertical component ofthe 778- AL T6BRN en i. makN D.6SkN tion 4 A prsalic Ven & lng Fred atthe lesa’ and samp “Hip petted wf the right cod The beam casvics'a uniformly distibated Tad so eet a theaghoa ts mage ee: of the following rly gives the deflection at the right end due focsiccs inten see Sane x aaeoO Simpl port ed, Heer 11 Wheel slowing mst nsry gies te detenon ste gh Pet ae ee eae nk ocr oa oe removed z © asy/aet the ol ay gee act lowing mon encly gee rection at the simples kN 2 © 1300kN edt iy D. IoMiN For the structures shown in 07, AN-O8, fe AN, determinate, statically indeterminate fo the second eterminate to the first degree leterminate to the second Cegree structural Engineering Civil Engineering 24 May 2000 ‘and Construction Reference Vol. 2 25 ———— ee — : Situation 6 - For the spiral column shown in Figure RC-52: Situation 8 - The implementing rules and regulations of P.D. 1594 states that for 16. Wich of the fling meat newly gies the norcment rate in a jt price contac quit overrutm ex undeans of not more than cent, ifthe eccentricity s 150 millimeters. ‘Cotimates per oonor poy teat (ue. pay item arent 7 A at 1 20% of he ttl stn et of te cobee) and dott B. 0015s %) per minor pay item in the bill of quantities need 17. Which of the following most nearly gives the ratio of the center to center Change Order provided that the same is authorized by spacing of reinforcement to the column dimension in the direction of Authority far the contract. a ‘cantract prices fora project areas follows: A. Os Coss Quantity" Unit Price (pesos) Bos D. 055 ‘00 1,000 18, Using the Interaction Diagram, which of the following most-nearly gives the 200 600 macthiaen downovard load that the column can support, in KiloNewtons: 500 300 20 © 1750 =o 400 2850 D. 350 ; 70 2 Which of the folowing most nearly goes the foal contract price of the ‘A. PA7S000 © P5600, 'B. P2300 560000 gol orto et. = 25, Which ofthe following is not considered as a major pay item of the project. ‘utolromaecton, b= overall branes mem ee be tem tems 5.11.61 Torsion effects shall be inclu ear ard flexure where m UE |! Whichot the towing may not he covered ya Change Order: factored. torsional _moment Ty >S2LE = 32 y. Otherwise, otherwise ease in quantity from 200 to 150 for tem 2 factored. torsional moment ¥ Oh | B. Decrease in quantity from 2500 160 for tem 4 torsional effects may be neglected. © Inerease:m quantity from 180 to 200 for teem § "For members with rectangular or flanged sections the sum 22y D, herease i quantity From 500 4 600 for Kees 3 Ibe taken for the. component recungles of the gection, bt the avetengng ange whose in design Si Totexteed 3 tims the SITE Sections located les than a clstance from face of support may toe designed for the seme torsional momen Ty as Uhl compated #8 | duane i of 10 meters subjected 10 8 10.50 KiloNevctons/ mater ble ly braced compact sections is 0.66 Fy. The allowable the span. ‘compact sections. are being, with their respective properties relevant of this ‘A rectangular concrete beain 300'mm wide, 600 mm, and 5 m lang deep is ‘used to carzy’ the slab sysiem shown in Figure RC-12, The slab is 200 nun thick and carries a total faciored load of 8 kPa including ve orsional reinforcement is to be pravided on the web only. Assume /. = 2075 MPa, 419. Which of the following most nearly gives the torsional moment on the beam dus tothe overhaging slab in Nm nondé O72 a cw ony meant Osis . es symbol x*y means raising the quantity or expression x to the 20. Which ofthe following most nearly gives the value of the sum Tx? y, due to Pee ‘the component reclangles of the section, in 3. following most nearly gives the minimum section mogul a0. e015 (Gx), 3 meterst such that the mascnvum flewaral stress wall not be exceeded: ee ate i a aera * 21, Which of the following most nearly gives the value ofthe limiting torque Ts 0315 ve i 26. lowing most nearly gives the minimum motent of Inertia ‘A 20 oe such that maximuim deflection will nt be exceeded: Bs Diao © a0o104 < Boon? Civil Engineering peeiecersspesra SS ag gves the critical path ofthe project kerr BS ‘20. Which ofthe following gives the eariest stat of activity m in weeks: A C31 BS Bay FIGURE 51-23 “structural Enginecting Civil Engineering Reference Vol. 2 Solutions to May 2000 Examination 28 May 2000 £2 Situation (1 103) Analyze one side of the bracket: Ry Direct toad on one rivet: Figure AN-O8 ‘Stress = 74.4MFa i The most streceed rivets are rivets 8 andl C: wl, 1295Mra situa :2(0:124) = 1.367 KN = 1,367 N 30 Ray 2000 Spring constant aac P= kuscx Kurc (05) 1020.2 Kase = 20404.N/mn £2 Situation (7 to 9) to be in equifibrium they must for closed polygon (as shown) 641.5" ~21622)(64.5) cos 22 S778 sing” siny sin77® 0-50.70; 7-534 $20 30°= 75.74%; bey pe te" 4579 +5347 +p = 180%; B= 8078" Vertical component of 7.8 kN force = 77-8 sin 9 = 76:8 kN o=as7e 12 Situation 4 (0 1012) Lean Deflection at 8 when the simple ‘support is removed: p= ee $00 2 soy : Deflection due to unit load at 8 3, 201800/E1 : . 31 Reference Vol. 2. © Situation 5 (13 to 15) Figure AN.07, (PIN-ROLLER) Reaetiot Dems uations, E =3 ‘DETERMINATE Figure AN-08, (PIN-ROLLER) Although there are eree wnkwotons anid Wires equations, the structure is UNSTABLE becuse the reactions are concurrent, The all meet at the y= 375/500 = 075 ef 150/500 = 03, Seructurat Engineering 32 May 2000 ‘and Construction rom the Interaction agra SEL nats 6.908= 8951 MPa 075 P= $ 600)? 8961;Pa 2346 1 2 Situation 7 (19 0-29) Torsion octet i t+ 20010 tooveranging slabs T=86%15)073) 7=4kNm 7 aa | Of Eady = 3007 (600) anon & sey = 102000000 mm? | Eey= 102m 20 7-NEe ixty- SAE R075 cm c000) Tus 19746864,78 Nome = 19.75 Nm Ea situation § @2t028) ‘ay_] Price [Total [Percentage] 1001000 | 100,600 —| 15) 200—|— 660 | 120,000 | —~as.265 500_[ too | — sooo | Tosa 250-400 | 100,000 —| 2.05% 350_| 700 | 105,000 [2211 ‘Foial Project Cost P| 475,000 ‘Total contract price of project = P475,000 Minor pay item = Ttem 3 Item 2 (Mojr pay item) from 200 to 150: Dacretse = 80/200 100% = 254.» 15% Needs Change ord Item 5 (Major pay t Incrense = 50/200 = 25% > 15% Needs Change Order ‘vem 3 (Minor pay item) from 500 to 600: Ierease = 100/500 x 100% = 20% <25% No need of Change Order ——$—$— —— Situation 9 @5t0 27) T= 1171875000 ment = 0.001172 mt ‘The required section must have [= 0.001172 and 5, 2 0.00382 For HY 26 66, S~ 0.00124 / (0654/2) = 0.00579 (not adequate) For W/ 30x 74, 5, = 0.00146 / (0772/2) = 0.00378 {not adequate) For W 24 x 62, 5, = 0.00119 / {01618/2) = 0.00885 fadequats) Thus, the adequate section is W24 x 62 D Situation 10 (28 to 30) From the diagram shown, the critical pathis @-bed-p denen Duration of project = 65 weeks Eniliest start of activity m= a5 weeks Macnematies, Surveying, and ‘Transportation Engineering Seat Nor 34 November 2000, CIVIL ENGINEER Licensure Examination Saturday, November 18, 2000, SETA ON: Select the correst answer for each ofthe following questions. t foreach item by shading the box corresponding to the eter of your choreon te ansver sheet provided STRICTLY NO ERASURES ALLOWED, Use pencil no.2.only. MULTIPLE CHOICE 1, ‘Twenty-eight persons can do a job in 60 days. They allstart complete. Five Pecsons qeited the job at the beginning of the 10h cay. They were feinforced with 10 persons at the beginning of the 450 day. How many days was the job delayed? ‘A. S78 days © ASF days B, Ltd days D. 245 days 2. Point A is between points B and C, The distances of 8 and C from point A ‘are 1000 m and 2000 m, respectively. Measured from point A, the angle of elevation of point B is 18° 30, while that of point C is S* 15”. Find the difference in the elevations of B and C, Consider the effects of curvature ‘and refraction, ‘A. did m ©. 487 m B. 326m D, 524m 3. Bind the area of the curve w+ y2+ 6x~I2y+9= 0. ‘A. 135 69, units © 9259, units B. 12359. units D, 13839, units 4. Find the distance between the foc of the-curve 912+ 25y2= 18+ 100y-116 ~0. me C8 Bb Dn 5, A right regular hexagonal prism is inscribed in a right circular cylinder svicse height fo 20 cat The difference between the circumference of the Tircle and the perimeter ofthe hexagon is 4 cnt. Determine the volume of 6. lar base of base radius 20 cm. Find the volume of the solid if every plane section perpendicular to a certain diameter is an isosceles right triangle with one leg in the plane of the base. ‘A 21303 ce C. 18667 ce B. 24155 oe . 20483 ce 7, Whatis the area bounded by the curves yé = 4x anc x2 = 4y? A 60 C. 6.660, 3.5333, The lope ofthe carve at any points given ear Zend the carve passe. through | ‘Determine the equation of the curve. fa y-62=0 € 28 Bit yt 6t—0 a. integral of x dx / Gx? + 2) with imits from 0 to 1. A © 0203 B. 008 D. 0267 10. The area bouinded by the curve y = sin x from = 010 + = » is revol about the ants, What is the volume genctated? re "A. ZAKS cunts C, 452 eu, unis tints D. real ta taving vot area is fo be 8 iattinam, ‘bis to be constructed, required diameter of the c 5m 5 ihe Hpvretal gon di teed oa cee slat A eno el Sh ie er eens Cres ere ee © 628m D. alm ‘60 kph applies a brake and stopped at a distance of 30:m. friction between the tres and the rosd is 05. What is the u xccount earning 8% us. How much isin the account after 20 years? < P3.377,45236 ‘amounts to 450,000, IF the comm 1 break-even quantity? ©, 2589 D. 267 ‘Mathematics, Surveying, and 36 November 2000 ‘Tramrportation Engineering. 37 ———— sss int rectangular coordinate of & point whose polar © (652.431) D. (31,356) fof a cone is 18 cm and its ax is incited 60° with the the axis is 20.cm long, what is the volume of the cone? aes © 12sec 169ce D. 1689 ee fhe magnitude ofthe space wector 5-4) 982 Biss A line ina map wa drawing is equivalentto yw many meters C. 0.05 m D. 50m a A 40m long connect eo tangent grades of 46.5% and ison ovation of point 2 2 2 26 with consecutive comers ABCDEF. Ifthe bearing 25° E what tthe Benning of side FA? } eo Nas" s D.Ne : 27. The perimeter of triangle i $8. cm and its areas 144 6g, cm. What the nc RC so he erred cs? Re me © 5520m he BS D Sizan Bi D8 | November 2600 Mathematics, Surveying, and ‘Transporcation Engineering Solutions to November 2000 Examination m1 th the job = 28(60) = 1680 man-days jon = 4441 Reduce x2+ yt + 6x - 12y + 9 0.0 standard form: + 5a) = m5 Distance between fori = 2e® $ units Ear Ve (Arrant Ad) Aye Ann 0 Aa™ Y{A0)(A0) ~ 800 com? ar Using the fore ‘Areas ge a eee ra Getee Bee ee yee ne a ty se Mathematics, Surveying, and ‘Transportation mnginecring 40 November 2000. Gs m10 au m2 G=-0028 =-26% (05+ G) ou F= Peet =1,000,000 easea Fm 76953,09202 Givil Engincering Reference Vol. 2 41 —— mis Let # be the money in the account after 20 years, then: Present wor if F~ Present worth of annuity for 10 years. F= 73,127,540 To break-even, Cast = Revenue cost= Revenue = Accident rate = No, of entering vehicles XN; N= 892,891 vehicles = 61157 mas a= 716" 200 =x 005 0= SORE = 298 7.162 800 624m mi ito Sa. 420 1 = 025 m 12-y 12-025 ished road = 12—¥ 12025 9 mi357 ae 3 Sp = 001387 hed road =-1.357%6 Mathematics, Surveying, and ‘Tranrportation Engineering 42 November 2000 Actual error = 0.02 x 25/000 = 500 mm = 05 m, mn = 100-322 (= 100 ~ 3222) = 35.6 fy/sec fan A= 4/5, A= 3866" AncosA _ dsin3866°~cos3866" A+sinA — 300538.66° +sin 38.66" a ‘The sum ofthe interior angles ofthe ot = 180°(6-2)= 720 Bearing of FA =1N 35° W_ Arars seein ee T= 129), r= 497 cm 2 17.32) = 1469.13 cm! 49" = 1.045 jovemibar 2000 TS. Two pipes | and 2 having the following properties Pipe I: Length = 1500 m, Diameter = 860 a Pipe2 Length = 1200, Wis required (9 replace these ZO mt Assuring taal the fr ioe damster? epherie temperature oF over 8 relatively shore B. potable water D. sewage ee Figure 03, mz as Qe algi/gira 1 24= 05770 3 (0.00192 B77 aan (0-25 F/° (0.001) seman Qu Dynamic force, F= 2® y~ # Aga 5 8 vn 2g ore = 29H Dymmichne = # aya Dynenk treaty Af Summing-up pressure head from A to F in meters of water FA 419088) + 8-2) 7 aoe 0+ 08tye3-2-44x=0 Oh, = n= 11990 47 48 November 2000 a Solving for the velocity of the jet vm fogttng? = (5?+0? =5m/s Since the flow is continuous: as oF 7a ms Bh= Me Pu) (0= 1194); Re = 15.254 = 0), R= 15.26 EN R=098m nw o- 29H ~ po 7.92 ms Hilo = 24.066 0 Forthe oquivatent single, Ducharge-0 Head lost = hfe Ho 8.0826(0.015)(2700)0? Tao a, D= 0874 m = 674 mim = 24066 2 Mark on jr chokeon ICTLY NOERASURES ALLOWED. MULTIPLE CHOICE Sinan -A pret ee id ot ighwrape The approved agency. al cl repos tia es lle Bidder C - Pass'21g 557.08 Which of the. followi goverment estimate: ‘A. A82,262,700.00 ‘& 10 B12 ‘most nearly gives cb B. Select the correct answer for each of the following questions. nr shading the box corresponding tothe leter de. Use pencil no. 2only. lations of P.D. 1594 states that no award of der whose bid price is higher than the proved Agency Estimate the AGE. The allowable (AGE) or the A corer than 70% © Pas9,678,100.00 D. Pt64525,300.00 most nearly gives the value of the approved P76 436,200.00 D. Pas635210000 Situation 2 — A continuous beam is as shown i ‘moment distribution method and assumi Which af the followin the loads on member B member BC, in percent. Use the mo lowing most nearly gives the mament at B in KiloNewtan- Which ofthe following most nearly gives the reaction at Ain KleNewion ion 4 — A 5-kg block resting on a smooth surface is pushed hori (E-11. The graph of force P vers lowing most nearly gives the acceleration of the block ‘seconds in meters per second per second, Which of ha fotosing mast nearly ges the velo ive fecone in tetrs perfor Sere Yet ofthe Meck afer 2 following most nearly gives the total distance travelled by the Width of 300 mm and an supported over a span of 6 id OF 25 KN/m and & uniform live = 312 MPa, “Compression necessary shall be placed af a depth 80 mm from the ing, most nearly gives the maximum tension steel area fixed end moment at A due to for singly reinforced condition. f Structural Enginsering cease pte 55 Reference Vol. 2 54 November 2000 TH Which of the following most nearly gives the required tension steel area In square mullimeter, ‘A. 5800" Situation 6 ~ A propped beam is as shown in Figure AN-16, The moment ‘applied ‘supported end causes 6 unit rotation at that end. 16. Which o ing most nearly gives the value of the moment M in KiloNew ‘A670 ©. 70 B 6% D. 70 17. Which of the following most nearly gives the reaction at the simple support in KileNewton, 18, Which of the folowing most nearly gives the moment at the foxed end in KiloNewton-Meter. e a ‘A. 00 350 3 530 B.380 Situation 7 - A project has been bid out by the Department of Public Works and Highivays. The aprroved agency estimate (AAF) 500 mull poses, The rezulis of responsive bids are as follow: Bidder A = Pos0.234451.98 763.12 Bidder E = P204758,425 54 ‘The nplerening role ond regulations of FD. 139 states that no aard k contrad shall be’made to a Bidder whose bid price is higher than the ywable government estimate (AGE) or the Approved Agen (AAE}, whichever is higher, or lower than 70% of the AGE. ment estimate (AGE) is defined as one half the sum 0 re cverge of all responsive bids. For the purposes of determining the average of Bias, bias ox of the being dow 19. Whieh of the folowing most neo project in pesos. "500,000,000 © P5s0.000,000 100,000 ‘AAE/AGE, gives the maximum bid price for the ‘C, Pab0,000,000 1B, 400,000,000, 21 Which of the following gives the bidder to which Uke award can be made, A. Bidder D ©. Bidder A D. Bidder E Situation & ~ A circular timber beam 250 millimeters in diameter hus a simple span of 4m. The beam carries a uniformly distributed load of a (KN/m) including its own weight. ‘The allowable stresses are 18 MPa for bending o grain. Allowable deflection is 1/240 of the spanlength, £ = 00 MPa, 2, Which of the following, most nearly gives the vale of w 50 that the alwable Bending ores wil nat be txteded. Hin’ Conver: the eieuar to square ston having the same as: cn en De A of the following. most nearly gives the value of w ao that the rable cheniey fest Welln be exec ‘a0 3 ZB Which of the fotiowing.m ‘Mlowsble deflection wilfnat, Situation 9 - A simply supported steel beam 6 m long carries a uniform load of N/m and an anal Compressive force of 320 EN. The properties ofthe A= 14700 ma Section Modulus, 5. 1921 x 108 mm? of the NSCP, for members subject to axial compression and bending —— + fi cy 2 © 06, 5 smputed axial stress, MPa y= yield strength of steel= 245. MPa pte " 6, Which ofthe fellowing net nearly ges the computed ial tes in the sam due to axial force alone acting on the beam, in’ MegaPascals. AD eet - D. 82 B77 26. Which of the following most nearly gives the computed bending stress in the beam due to the unifonn load alone acting, on the beam, in MegaPascals, AL ot C70 Bt B. 5 27. Which of the following most nearly gives the value the interaction equation, AOS C07 Boa B. 06 56 November 2000 eee Figures: (AAE + Average of responsive bids) 000,000. + 464,525,302.47) = P48, 262,651.23 E = 20,000 res Tah i ant Figure AN-16 “END d seruccural Eneinecring | Civil Engineering Reference Vol. 2 59 Situation 4 (10012) F=20N ‘Ferontne dagram ‘Moment at B =-1287 KN-m © Situati 09) versus time. But since accelerati diagram can easily be construct tds t dy tAy (90) *80(2) + (2/3)(2)120~ 80}+ 120(1) S= 413,33 m 180" + 0~ 23089" 37455 Number of 25-mm bars = 3 305)? = 763 say 8 ) = VEGNORY(S)= 0; M=4R >) he er than. 70% of AGE (P337,583 855.86) cant be made to bidder A. It nd among the two, Bidder D is water. diem Deflection: w= 575 Nf = 5.75 kN/m g Structural Engineering ‘and Construction Civil Engineering 63 ‘Reference Vol. 2 CIVIL ENGINEER Licensure Examination Friday, May 14. 2001 INSTRUCTION: sel Morkonizene ans of your answer sheet provided, ICTLY NOERASURES ALLOWED, Use pencil no.2 only. ‘MULTIPLE CHOICE on. 4% upgrade suddenly applies «brake. the the ties and the pavernent 3, haw far ing the brs? 2 jume of 1000 cubic meters, 12 m diameter on top face rising when the water is 12m A. 0.231 m/min © 0828 m/min B. 0712 m/min D. 0566 m/min 5: What is the period of the graph y= sin 12 An 6 Am openta sheet having an area of 20 ight, what isthe height of the tank? 7 Anequ fe expectancy of 10 years etining balance method, 96480 B. Pi47as600 Mathematics, Surveying, and 64 May 2002 ‘Tranepareation Eaginecring 65 ———— $$ <<< i ___ 5 Fig teffrive rte on a nominal ate of $5, susp ont wumscribed about a tangle whose area le 48.23 C 85m the triangle measure 18 cm, determine length of D. 1012% eof the wiengle Ine r= Ay +7 =D from the point (2.5)? ©. 564m 5S 2 co D. 234m vey, a stall gue reading of 8.15 m was observed a the 10 of the sounding was 176m. The zero mark of the staff ion 148.2 m, Find the elevation of the point where the de © 136.25 m D7 75m Tre sum af et tems ofa series 995 6 Find he ith em ofthe u rea bounded by the curve y= 4 series. ¥ | AL 1434 C1458 C. Ssquare B iaa2 Di 142 ‘D. 6 square units Bo fone measure 14cm each, What is the maximum possible 12 ugle? © Stem? D. 9B em? of (2-8) / (2-2) asx approaches 2 Tie fot second and third quadrants of the curve 2+ 92-9 = 016 revolvedabou! the line +3 = 0. Find the volume generated. 1G 37838 cb D. 278334 cub B. square plate ABCD of uniform reed by 2 4x1 + Oy? = 14K having equal igs The ist sring is located at ated 15 meter, Whats the equation ofthe diameter? cm from 4 along se AD, andthe thirds at the ide CD. =0 What percent ofthe load is carried by the fst and second supports? A box 50% B 60% De gts of long chord from PC to PC i. chord from PCC to PT, La = 178.23: hord fromPC to PT. c Mathematics, survering.an@ Civil Engineering ‘Transportation Engineering Reference Vol. 2 67 bie Solutions to May 2001 Examination ma. Sennen Marhematies, Surveying, and ‘Transportation Engineering ER=c~1 = ct ~1=0.08572= &872% factorable by 1 ihing kph, + 6/60 ee Mathematics, surveying.ané | Civil Engineering Yransportation Enginecrins | Reference Val. 2 = 13265 m maa = 40215 m 72 May 2001 a0 mn 242)0=2) rary slime + )=2+2"4 n= 148.2 815-175 n= 138.75 2 1m of first n terms ~ Sum of [3°2- 6] = 1458 mas aos (2-19); = 31 mymnin 74 May 20021. R= ME Ris O3W= Re = GOK 1e weights carried by the first and the second supports. Ma 51 Hyarautics ane 76 May 2002 Geotectinical Engineering —$—$_—$—— Sen Nor 12:00 p.m. = 06.00 p.m NOERASURES ALLOWED. Use pencil no. 2 only MULTIPLE CHOICE as mife: The slope in GID. 11 the nod mining seepage flow, the paths of equal C. flow lines: ey Setar oe deep end palit spillway discharging atm deep and a crest length Sel mk, 882 tnd he dachange ee Necuerepersecond. A 8630 Kinematic wiseasity of 15 x 107 m/s flows though a 75-mm y of 0.8m/s. Theflo 15 carries 25 f0/s of water at a don a slope oF 0.0002 with n = 0,015. ‘normal depth of the canal in feet volume of soli ie ica dinmcter of2foxt with rough lope of the energy gradeline is c 578 B. 1458 = 1.2 m/s. Determine the total head cas coefliclent of 0.013 flows 6.0025, determine the discharge cd to hald the gate in position. ©. 46737 Ibs D, 50885 Ibs CONE PENETRATION (HO) ‘eight of « 12slug bould ue to gravity 1890.76 ft/s e Liqurp Limrt TEST oo os ws ‘Figure SH D= fmm Figure 23 80 May 2001 ‘Geotechnical Engineering 8 $10.06)" 0510) | Dehra V= 17.08 mys, 825 = = 185902001 | az yextnd: 20," cos? 0 15m Pet = Pome 6862/5 boot sais? 1S=xtano- a 216862)? cos? OF 2 594m =379m He ms 035 | 23-0538 1-035 ; tay AZZ (62.4) = 110.36 pet } | 2.65 1045 ; (624) ~ "M40 pet OA 100% = 96.24% p= By k= 981(08) + O81 <082)(065)~1267kPa | 82 May 2001 on 2" 356 mys ais Q= Co LH = 3.2(10)2)7 = 905 mys mu e- 22 . 087s) = 40000 > 000 urban) v 15x107 a _ mis au | Nydrauiics anid | Civil rit onl ates BE itesioerins as ‘Cone Penetration (rum) PLASTIC LIMIT TEST A)(S) = $64 5KN F,(38) = 36827028) Fp=20.2kN 2302 (65(1.2) = 78 kPa (OK) =F 3645 230.2~ 134.3 kN m2 Me LAY ZOOL Myarautics ana Geotechnical Engineering Civil Engineering 87 Reference Vol. 2 Seat No: CIVIL ENGINEER Licensure Examination, (08:00 a.m. - 01:00 pm. STRUCTURAL B CC ICTION SETA cortct answer for each of the following, questions Nak onl one item by shading the ox corresponding to the letter irchatceon the sinner sheet provided . ICTLY NO ERASURES ALLOWED, Use pencil na, 2only. = A contractor was awarded a fouryear contract for trucking Fe wishes to purchase pick-up truck worth PI 000,000 each. The be used for 2,000 k fuel consumption le all the in Figure AN-26 is 150 mx ug stress on extreme fiber ie 8 M Aaig eres Cae wis oat fant been cB D. 215 6. Whichat the following gives marian Yalu o i KioNewin, A125 c m8 B m0 ©. 180 Situation 3 A square tied column is to carry an asial dead load of 500 KN and tmlakaive bad of ADEN" Assume [> 21 MPs ond j= 275 MPa. Use 8- sin bars and reinforcement ratio of 2 Structural Engineering ‘and Construction 88 May 2001 ring gives factored load P, that the column 2 B 1845 8, Which of the following gives the required A Ba C367 B. 392 D. 4a 8. Which ofthe following gives requied numberof 29mm bas oar) Bo D.10 ced concrete beam having & width of 280 mm and a 20 mum is reinforced for tension only with 5-28 mm bars, = 2 MPa, f:=0.45 f,n=9, Use working sireny 10, Which of the following gives m of the neutral axis from the outermost compression concrete. A. 237 cus B. 198 D276 11, Which of the alements is true for the given beam: A. The sect the code B. The tens elds first © The con D. Thest ds simultaneously Se uuanGolleera byes ta ares: areas eh Gees KiloNewton-meter. A 138 C18 Ber D108 Situation ~ A certain equipment hac fist cost of P900.00, life of 8 years, and 00000, 6 ig gives the book value of the equipment after 5 years ine method e BE roas0000 BPs, 200.00, 1. ring, gives the book value of the equipment after 5 years year sedigit method © P257,329.00 B516,667.00 15, Which of the following gives the book value of the equipment after $ years ‘using the double declining balance method. ‘4, 325,582.00 C. P2s3,235.00 B P245,389.00 D, P2igs74.22 Situation 6 ~ The magnitude of a force is 1000 KN and passes through the origin ta poing 324) 16, Wheh: lowing gives the X-component of the force in KrloNewton Aad E3932 B 4535. Dam Civil Engineering Reference Vol. 2 89 17. Which ofthe following gives the 1-component ofthe force in KilaNewton ae C'ssr1 3. a327 ing gives the Z-component ofthe force in KiloNewton ¥-13, the product of the crass 000 Newton. 20. Which of the following gives the ‘unit load at C, AY © 125 B. 075 Dis 21. Which ofthe following gives the vertical deffecion at €in mum, A535 ©. 845 B 675 D335 Situation 8 ~The angular set Piste, Both mateals re A ress is Def, The shearing te Properties of 150% 90% ‘hres = 2751 mm? 0mm fmm 22. Which of the following gives the value of a bin millimeters A. 267 Car B. 399 D. 351 25. Which of the following gives the value of iin millimeters, A 287 cr 24, Wh Phe tf the following ves the value of Pan KGNewton c se 26 Which of the following gives the Iocation of the resultant load from the left apport, in meters. A 285 C395 Bis D. 375 22. Which of the following gives the reaction ut the left support in Kilo A 30 eae leads Structural Engineering, Civil Engineering 90 May 2001 ‘eas Contraction Reference Vol. 2 91 Giiation 30 ~/A simply supported beam has he Gums oction shown In Figure oT 187 ASG fs Dek 101 30 iM ISHN kN ‘bending stress for laterally supported compact rwable shearing stress is OF, Allowable deflect LSM ismasmlism! ism Figure AN-36 jon is adequate for deflection only jon is not adequate for flexure, shear, and deflection Figure AN-13 May 200i +OM-+ annual depreciation ,000(0:15) = P150,000 (1,000,000 —300,000)(0.15) +051 Bhiinthe load diagram: Mc =-18 = 0.02 Ay = 0:92(159,385.2) ~ 8068 mm? $20). = 3068:N'= 98say 10 ©) Situation 4 (10 to 12) Aum $0525 4A,= 30788 mm? Ay 27,709 ment Ren 277091520 ~ 2) 140 c+ 27709 ¢ 14,408,680 = 0 8mm c= k= k(620) = 2368 k= 04554; 7= 1-1/3 = 0.8482 ‘Thus, th moment capacity is 1388 kN-m From the results, the concrete yie with the code, st and thus, it does not comply 94 May 2001 2636) = (900,000 - 200.000) 2 ot = P583.333.35 rau STAs |) Pasras amnaeo = a F200,000_ 3 700 This the hook value ater S years is P284, 765.625 ote Since the salvage value is P200,000, the minimum book:yalue must bbe F200,000 and thus, the depreciation charge after 6 years must not (Gxceed P13 574.215 Reference Vol. 2 95 eS Using the formula: sv, =ec(1-2)"»sv riieanan(s-2) BV sm Pais,s7.2a ©. Situation 6 (16 to 18) Forve = 1000) through (0, 0.0) and (2,3, 4) Le y(2-0)*+(3-0) +(4-0y = y2B- B=W os 199 2=2 Fy=Forcex BHI = 1000 2-9 55709 4 rs ry Fos Force x = « 100042 = pap: T ie ee 2 Siuation 79402 veto actual load: (5) = 400 500 N (compression) 600 + 5003/5) SOON (tension) {compression} Use 1.25(9/5) = 0.75 (tension) 96 May 2001 tion = 1/360 = 16.7 mm F, = 0.4(248) = 99:2 MPa sin = 2810 os ~ 107.14 MPa > F, (not adequate for shearing) F, = 0,66(248) = 163.68 MPa = 20 (6) 10(45) + 30(8)+ 15(L5}; Ry= 254N “Ths, the seetion ie adequate for deflection only Mathematics, Surveying, and (Civil Engineering 98 November 2001 ‘Teamsportesion Engineeting Reference Vol, 2 o : ——— ss Seat Nes 8. Water flows into a tank having the form of a frustam of right cieubr ‘upper radius of 15 m and the lower radius 1. When the water in the tank is 1.2m deep, the surface rises at the of 0012 m/s. Calculate the discharge of water lowing ito the tank CIVIL ENGINEER Licensure Beamination Saturday, November 17,2001 08:00 a.m. -01:00 p.m. A002 © 0.08 Marea C5, SURVEYING, AND TRANSPORTATION ENG’G, SET. B 005, Do IEMATICS, SURVEYING, £ 9. Using a 25-m tape, a square lot was measured and found to have an area of F each of the following, questions, Th If the total error in area is 4.004 square meter short, what is the the box corresponding to the letter error in each tape length? coon the answer sheet provided, 4: D5 m oo sere S e006 too ong ise pencil no. 2m 0.008 m.to0 short 0.005 m too pe cip yee j 10. Tangle ABChas side AB = 160m, BC = 190.em, and CA" 190 cm, Point D ‘salong side AB and AD = 100can. Foint Eis along side CA. Determine the ee oe length of AF the area of triangle ADE is /5 the afea of tangle ABC 1. Determine the central angle of a 350-m simple curve if the nearest d ‘A. 1653 cm © 1824em from the curve to the point of intersection af the tangents is 18 m. B D. 1738em ers C36" TL A house and lot costing P2 million was bought at a downpayment of D, 56 'P500,000 and PI million after one year. " balance wall be paid. 24%, compounded seini- 24.00 15.00 ‘be made using the digits B. PL 12 How many from oto joning of PRC (point of reversed curvature), Use ore D. 430 © 67 +4673 ate the integen! of » cos (4x) dx with lower limit of Oand upper limit of B, 67 + 689.23 5) /Aoagh bos an open top 030m i jeeal wrangle 30 oe on os e Volume of the water in eu.m. of 5% from STA 12+ 180 to STA 12 + Ca - i apt of he radwny 4 fil Ate B 005s D. 0.065 ‘other station, the center height is 28m cut. Find the grade of the finshed 6 Whit isthe present worth of a perpetuity of 710,000 annually is money is road, A 2% 27% B Les D. 3% 16. The observed interior angles of a triangle and their corresponding number of observations is as follovrs: Comer “Angle No.of observation A Fi 5 B 5° 6 c 7 2 oath inyoanded month? 100 Nowennibar ZOO _—"“anepareston bastneerint "Determine the most probable value of angle C. ea eee B. 73°54" 32" 17, The cost of producing 1 commodity consist of FSBCD per unit far Iabor and material cast and P25.00 per unit for other variable cost. The fixed cost ‘month amounts to 7700,000, If the conumodity is sold at P290.00 each, {isthe break-even quantity? A. 3200 B, 3500 i 18, Determine the moment of inertia of the area bounded by the curve x3 = 4y ‘the line x= 4 Qand the X-axis, with tespectto the Y-axis A512 C521 Bl D715. 19. Acar travels ona banked circular curve, The front wheels of the var are 1.5 m apart and the curve is banked such part. What should be the minimum radius of the curv 30 ‘no lateral pressure on the car’s wheels ata speed of 62 kph ‘A. 2675 339m, D, 252m, Find the volume of the solid in.ce jwen diameter is an equilateral B 238 D. 2638 walls from his house to the office IF he the rite of 2 kph, he will arrive 3 minutes ‘walk st3 kph he will arrive 6 munutes lato house to hie office. © 24km D. 14k 2A°= 2B + 3C = 28 a 24, A told stool ball of radius 3 cm is immersed in a cylindrical tank containing, ‘note toa depth of 10 cm. Ifthe water in the tank rises 225 cm, what i the lameter of the tank? Reference Vol. 2 101 Tis te sd ofa tnngle emse Socm 18m an Aca Wha he length of ve meton dawn the longest sde to opposite vertex. 12325 an e038 11S em B, 13005 an ‘A-circle of radius 9 cm is circumscribed about a triangle whose area is 48.23 juare em, If one side of the tangle measure 18 cr determine length of theother side. ie p A. 437 m ‘09m fan ellipse are 4 and 6 uit Deteranine the second! eccentricity of the ocr ‘A. 0204 0508 B 0305, Bi. oa0a ‘The sum of two numbers 5. What isthe minimum sum of their cubes? A, 9/3 c 8/2 B ya D. S/5 Water flows at the rate of 16°m?/min in conical tank 12 m diameter on top and 24 m deep. When the water in the lank is Irmetens deep, the surface ts rising at the rate of 0.566 m/min. Find the value of f A. 10m Cam Bid. Dam en ‘ANSWERS: he 6A wo 28 ATG ZA WD Das AO WA B30 A D8 BC e306 Mathematics, Surveying, and 102 November 2001 ‘Transportation Engineering, Solutions to November 2001 Examination ‘The nearest distance from the curve to Pl is the external distance, E m2 RC = Stationing of PC. + Les 2) = 160m os Gy - 656= 0 ‘iy! = by + 9) ~ 656 + ABCA} + 369) = 76K Reference VoL. 2 103 mr ma ‘The water will assume the shape of a triangular prism The volume L, where Ag i the base area with the shape of equi 1292-1; 4= 010471 19,900 O10871 = 95 499.07 From B to C: (a= -10 m/s) vieart2aS 3 = S1+S:4 12 6667 # 2489 +12 =103.36 m 208 ka a= 025% Reitx=14 125% PH Rae Ret va Beara y= Migs arp v= F jooissasios oa7sine 3 Civil Rosineeting 105 mn Number of combinations i‘ fee rom fits =10- foae usa dunds; do=costedy v= 106 November 2002 Mathematics, Surveying, and “Transporation Enginerring fren ree ae f Baia a q In Triangle ABO we Hinda0> sin 1 x= 92542 m Intriangle ACO. sin 30° = "y= 92542sin90"= 4627 m ois) sie 1smy_ 14-02 | 0 finished road = Greet = 0.02= 2% ‘The weight of error of each observation is K/N, where K'can be any AN ts the number of observation. The ideal value of Ks the allthe number of observations, 12. K = 30 The error must be subtracted to the observed values since the sum of the observed values is more th 108 November 2001 ‘Transportation Engineering aw ‘Such that there willbe no lateral pressure ‘onthe wheel tan @ = = 109 to office, Jand walking kphy he will arrive 6 minutes late, ofa = 136/5 30) 10 = 3¢- 126/5;t= 91 TnFg, 2S =349.1)- 126/= 21 km By Cramer's rule: t famstnem in Mathematics, Surveying, and 110 November 2001 “rrantportation Engineering a -2 2a] 2-2 new [i 3-35] 13 a a ols 4 Neo [12+ 90+ 96] - [216-120 +4) = 74 Ne 2 74 2 =a eo Sum=sA+B+C=4-5+2°1 ca Vigtere = Vitaptaced $10 FORE D-San as e rev 2nmd | min eS onin “reo sec o= 2 rad/s y= (din/15)? (2) = 1405 ays? 26 Solving for by cosine law 8 inuiangle ABC 24! = 264 18- 2(96)(18) 005 0 0 = 36.336" s 18 Je BCD: SHEET -20908 conse" g a Civil Engineering Reference Vol. 2 iii wa ‘The diameter of the circle is 18-em, hence one side of the tangle is the diameter of the circle and by principle this triangle is a RIGHT ‘TRIANGLE Ars thab 4823— 4608 ab=946 3.0) areaip @=32-b >@) Squaring both sides af equation (1) mB 58045516 (2k fF) = SB0KE5I6 b= 3262+ s3045316=0 P=31848, 6-568 a= 17.092em Thus, the shortest side is 6.64 em, ea 64am 5 webs BB+ 1b= 4.009 Second eccentricity, <= £ Pier 1 = 020 é- 139 (OTHER ELEMENTS OF ELLIPSE: Fat ecntniy.e= © tips amass, j= 28 “Angular eccentricity, a= £ Second flatness, f = at Mathematics, Surveying, aad ‘Civil Engineering 113 112 November 2001 ‘Transportation Englacctieg Reference Vol. 2 ‘tp SeatNo Let cand y be the numbers, then SETA INSTRUCTION: Ssléct the correct answer for each of the following questions “Mark gnly-one anewer for exch itern by shading the box corresponding tothe leer STRICTLY NO ERASURES ALLOWED. Use poncilno.2 CTLY ss pec Sum = (6/2) + (5/2) = S pencilne. 2 only MULTIPLE CHOICE 30 eek 1 Pat ge ng cg ofS erent ; The channel al on Slope of O00 rah re GTS See Smee are Ae 525m Ey ues berecrer toes calc ae Gay wa hea pr ead Wa Seton ee eaternael vath of the weir. 3. The section of a dam is as shown in Figure 013. Assume hydrostatic Determine the total reaction per atthe base ofthe an 16= 32 j2(0.566) spi (0560) h=iam vty dam 4.2 m wide and 25 m ream side, What is the factor Lf and use unit weight of concrete ~ 23 C08 D. 09 8. The two reservoirs shown in Figure O14 are af 900-nm pipe with roughness caf ue of = 0.024, What is the percent rec A 50% C 73% B 25% 1D. 200% 6, Determine the value of y in the manometer shown in Figure 015 ‘A. 0432 m C0203 m B. 0.234 m D. 0334m 7% A 18m wide rectangular canal carries a flow of 2.4 m/s. What is the critical depth? ‘A 0569 m C. 0452m D.0378m. a mpl ken from the ste weighs 8505 grams. Arte oven cd S94 grams. If the specific gravity of solks ts 272 ‘void ratio of the soil, © 187 D. 1432 ydesuites ane Civil Engineering 114 November 2002 (Geotectndeai Enpincering Reference Vol. 2 115 eine taken from the site weighs 850.5 grams. After oven 18. A semi-circular flume 1.2 m in diameter is ‘ay flow of 1.75 ant/ eying, # weighed! S944 grams. If the specific gravity of solids is 272, when flawing full. Lf the roughness coefficient » = 0012, what slope is determine the degree of saturation of the soll. required? A789 C0933 A 00009 © noo B 0979 D. oss eee 11. The section of a cofferdam is as shown in Figure 17. IF the coefficient of ). The discharge through a Cipollett weir with crest length of 8 feet under a permeabiity ofthe sal ts = 5» 10> m/s, defenmine the seepage imo the fad of fete rarest to! itches: 1 colferdam. A. acts AO. © aa0tmy7s B. 154 chs . B. 0.072m 1D, .058m%/e ‘2 Water flows through a horizontal Venturi meter whose iniet diameter is 31 TL. Water flows through a 25 cm diameter nozzle at the rate of 10.L/s ‘cm and throat diameter is 19 cm. ‘The pressure at the inlet i 755 KPa and at Calculate the dynamic force available at the nozale tip the outlet is 550kPa. Determine the discharge in m/s. Neglect head lost, AIBN C32N A073 C089 BL 276N D. B a97 D. 062 12 Water flows through a horizontal nozzle Jasge tank under « constant head of 3 m. The n 21. An undisturbed soil has moisture content of 54.2% and a liquid limit of 56%, Determine the compression index ofthe si the ground. Determine the time for the water to reech the ground after A, O65 © 041s leaving the nozzle B 02s ‘A: 102 Cas 135 consists of solid particles, wat B. 078s D. 2873 ier and air is called voids and 13, The flow of water through a Cipolleni weir is 24 m/s when the head is 0.6 es i call amy, Determine the crest length in meters, ume of soil is terms ‘A, 215 C355 dry unit . moist unit weight D. 185 dry density D. moist density 14, A spillway controls a reservoir 5 hectares in area. ‘The permanent crest is at 23. In wastewater treatment, the process of removing fine residual suspended elevation 75 m. be drawn from elevation 765 m to 75.5 min 42 i known as: ‘ainutes. Pind the length of the spillway sams, Use Francis form ‘A. Aeration © Soda ash ‘A. 157 C108 B. Filtration D. Zeolite 13 m below the ground surface. “The clay has o unit weight of 16 KN/m? above the ‘and 30 KN /m) below the water table. Determine the total stress at the bottom of the clay layer. ‘A. OO KPa c akra D. 98 kPa: ae a specimen of saturated (normally consolidated clay was one dor camber confining estar of 90 KilPascals, The ‘nial stress on the specimen was ten increased allowing the drainage from the specimen, The specimen fails when the deviator stress is 0 KaloPascals, ‘The pore water pressure (U) at that time was 40 KiloPascals. Which of the felloang most neany gives the consol date drained fictonangle (ph) ‘422 C 34592 B. 2678" D193" V7. A 2m wide rectangular canal carries a flow of 26 m'/s. Determine the criical velocity in m/s. A321 ©. 158 B 234 Bi, 278 -END= Civil Engineering Reference Vol. 2 ais ees 18. A semi-circular flume 12 m in diameter ls used to carry aflow of 175 m/s ‘when flowing full. If the roughness coefficient » = 0012, what slope is required? B. ons B. 00017 19, ‘The discharge through a Cipollet weie with crest length of @ feet under a head 0f 35 feetis nearest to: ‘A. Wocks © 197 B. 15tets D. 23 fs 20, Water flows through a horizontal Venturi meter ‘mand throat diameter is 19 cm. The pressure att the outlet is 550 kPa. Determine the discharge in mi A. 073 C089 B. 097 D, 062 21, Anundisturbed soi has moisture content of 54.2% and a liquid limit of 56%. Determine the compression index of the A 0165 C. O41 water, and air. The combined volume its ratio with the volume of solid The ratio of the mass of solid to the total ‘A. dry unit weight B. dry density D, moist dens 2B. In wastewater treatment, the process of removing fine residual suspended ‘A. Aeration © Soda ash B. Filtration D. Zeolite Figure 045 Figure 018 een Girl Eesinerics 117 Am 6d + Y5(D(1A2Bd)o2 Anéd+1AaRe P=6+ 20.7430) Pa6+346d pe A a Sdetaned BP Ga3.486d Q-A2 RASA By trial and error using the choices, ¢= 2m Q-G Line 66 = 1.56 5 (1.2)? L=52m. am fren? Consider L-foot length of dam: 10+40 3 (6O)(1) = 224,640 ths .s¢242 550) 208080 FS =07 Hi=20m=17 ye 102%ntL0? pers Atyeard: 2 n= ston cg Atyear It: jp 1025002 Dy 420-3101 m/s Mi"). y= 155:m9/s Civil Engineering Reference Vol. 2 119 ae m7 as me Summing-up pressure head from ‘AB idole of water a vig 2 where ps=0 y-034 m = ae 8 @ 24 =O. 1309 m/s per meter wi + St = 1.259 m/s per meter width den = 05659 m GeGme eta W | s505 aie e177giee ive, aa Wo. 8305~ 5044 c= We. = = Wey saa 17 = BAS2THO) ),. 01.396 Tre a yoo 4= Gx 1099003 = 0065 m/s per meter Since there ae two ao identical sides p= 0.083 = 2 = 0.086 mys per meter Peat y- 2 -_ OMl apale E A 710.025)? (20.372) = 203.72. Qe isso Li 24-1359 Los? L=278 m fees | Cot | Tie Ay D{10 acme asm ~ 2150.000) Ta, Civil Engineering Reference Vol. 2 ee ae 7 You at Yee + Ys Pr COQ) + 16(1) + 14) pre 8d kPa or drained Condition, os = 90-40 103 = 50KPa Deviator stress, 01 = a3 = 60 kPa 20 sind= 55 ¢=22000" 2028. oe d= fF wossen soos Q= A= (aye 26 (20.596) 9:0 = 2.338 mys Q-Av-AL Rn sv2 1 175 $0.2" So (2/479 542;5 = 0.0017 L=B ects (iu / 328 0) = 2439 m Q 18591 Hi = 1.959(2.499901.067)9 Q=50mi/s = 1%63cfs “Energy equation between Dan: fiw 4 nen 2 3 YyAy Eye a ti ag fe on C= 0.007{LL - 10%) For undisturbed soil C= 0,009{L1 - 10%) G = 0.000(56-10) = 0.814 Civil Engineering 123 Reference Vol. 2 Seaton CIVIL ENGINEER Licensure Examination Sunday, November 18, 2001 (08:00 am. - 01:00 pm. STRUCTURAL ENG) INSTRUCTION SEPA INSTRUCTION: Sslect the correct answer for each ofthe flowing questions Marx oniv-one answer for exch item by shading the box corresponding tthe eter of your chaiee on te answer hoot provided. STRICTLY NO ERASURES ALLOWED. Use penil no. 2only. MULTIPLE CHOICE, Situation 4 ~ A cylindrical depm 2m in radius is held by a rigid bar AB hinged cable BC as shown in Figure ME-SI. The drum seeighs 1500N. friction in all contact surfaces. 1, Which ofthe following gives the normal reaction at D in Newtons, Ae) ©. 3000 B. 3500 B, 2000 2. Which ofthe following gives the vertical reaction at A in Newtons. A M68 ©. 1259 B. 1853 D, 5642 3: Which ofthe following gives the tension in the cable BC in Newtons, A 27683 © 2438 B 2554 Di, 2541 Situation 2- A renfosced concrete rectangular beam ha awit of 300 mn and fm effective depth to bottom bars OF 20 man. The beam elnorced wih fic Sze bottom bas and two 2mm top bas lead 6am from Top Of the Beam." Concrete strength f= 348 Ml and sel strengin f= 6 MPa. 4. Which of the following gives the balanced reinforcement ratio of the beam. section in percent. A412! 53% B. A723. D. 439% 5. Which of the following gives the depth of the compression block in nullimeters. ‘A. Hal C156 B73 Di. 125 6. Which of the following gives the nominal moment capacity of the beam in KiloNewton-Meter, ‘A. 5708, C. aad B 743.6 D. 4238 Situation 3- The implementing rules and regulations of PID, 1594 states that for unit price contract, quantity overruns or underruns of not more than fi major pay item (be. pay item ‘which represents at last 20% of the total estimated ces ‘owenty-five percent (25%) per minor pay item in the bil contract) and Structural Engineering ‘and Construction 124 November 2001 ‘ot ke covered by & Change Ovder provided thatthe same is authorized By the Approving Authority forthe contract. ‘The quantities unl contract prices fora project are as follows: i ‘Quantity Unit Price (pesos) 200 750 20 70 100 850 150 00 300 50 20 500 750,000 ©. Pa 000 B P730000 DB. Psi0.000 8. Which ofthe following gives the minimum price af an item in onder for itto ee serpy pparalel to gratn Fin Table 3.1 Short columns (1/d-of 11 orless) Fink Intermediate columns (fd greater than 11 bat less than Reference Vol. 2 125 ings! at th en with FT properties of come Pilppine woods Bt 80 sree grade ae a follows: Table 31 gives the classification of the column, © shortcolumn te column, D. not approved by the code following most nearly gives the value of the allowable unit compression parallel to grain adjusted for f/d rato, in ot © 321 B86 478 12, Which of the following most nearly gives the axial load capacity of the column in KiloNewtons A765 cus B m3.2 D3 Situation 5 - A dump truck starting from rest move up an 8% grade at a acceleration attaining a Velocity of 60 kph 75 x from the starting, point. The truck weighs 1800 kg with its center of gravity 050 m from the me front ana rear wheels. ‘The front and the gives the minimum coefficient of friction between the tires and the road that will prevent the truck from sliding while iis at following, gives the normal reaction at the front wheels of the truckin Newtons, A. 9432 © 5220 B. 10248 , 811 Situation 6 ~ Section 585 of NECP states the following about control of election S951 Reinforced concrete members sl to flexure shall be designed to have adexuate stiffness to limit deflections or any deformations ‘structurat Engineering 126 November 2001 ‘and Construction Thataffect strength or serviceability of a structure adversoly. 5952 One-way Construction (Nonprestressed) 595.21 Minimum thickness stipulated in Table 5.95 (3) shall apply for one-way canstacton not supparing: or attached to patitons oF other Construction likly 0 be daaged by large detections, ties computation Of deflection indicates a leser thickness can be used without adverse effects ‘TABLES. (a) - MINIMUM THICKNESS OF NON-PRESTRESSED BEAMS (ORONE-WAY SLABS UNLESS DEFLECTIONS ARE COMPUTED ~ ‘Simply | Oneend | Both ends supported | continuous | eontingous Members not supporting ar attache partion ax oer ‘omsracton ily toc dama pe Py ge deistons Solid onenayalats |b /20 ae tps | Lies ‘snes slabe ne © Spartlengi isin millimeters ‘afc gots shall be sed cureely for members with normal clensity 2500 bya) anat grade 415 reinforcement For other conditions the v ‘modified as gh sphere kara el music 2100 ) Tepat ie sataes alle eegpacs Oy Qt65 ous, but ots th 103. Fara ants Me, fac shall be lid by ‘The section of a solid one-way’ slab is shown in Figure CO-37, The concrete is a normal density concrete with a anit weight of 2550 kg/m). The reinforcing steel used has a strength off, = 276 MP Mick of the following gives the ‘minimum thickness of slab By in rullimsters 4. 400 <8 1B, 353 : 7, Which of the following gives the minimum thickness of slab B: in millimeters, Ae © m2 B. 243 Which of the following gives the minimum thickness of slab By in millimeters 2 A. 216 i. 250 AB 199 B. 236 127 Tid steel shaft 60 mun in diameter ond 65 m Tang is subject @ in Figure ME-29, The shafts atached to a 1ighd support. ie gives the reaction atthe rigid support in. N-m, Cc. 1200 20. Which ofthe ol lege tae oe the shat ic the Felling pel tig wncietvm soesring sree tHe ah MegaPascals, ork : AO 28.29 C036 baa 52s 21. Which ofthe flowing pes the tot angle a vs at he fe end in ae come mize bias Situation ~ The lator and material cost fora viprapping work is as follows: oe Niprapping 1 Foreman @ P30.00 per hour 1 Seiled worker Pi 00 per hour 5 Unsilied workers @ Foo (00 per hour ©, P678.00 B. P5900 D, P5210 2B. Which of the following, gives the cost of materials per cubic melee A. PS65.00, °C, P7500 B. Paes D, Pss5.00 lowing gies the loa eat af prepping pr euble meter ma byssz00 Se crab 754,00 D. P1435.00 A propped bears i az shown in Figure AN-16, The moment ‘applied atthe simple supported end causns uni rtalion at that end 25, Which ofthe following gives the value of the moment Min Sm c. 60 B. 550 D. 500 26, Which of the following gives the reaction at the simple. support in KiloNewton. th Bee ee © 10455 B. 7585 D.9375 27. Which ofthe following gives the moment a the fbed end in kN-m. A. 280 150 B. 450 D, 550 128 Noyeaber 2002 ees Situation 10 = Section 45.1.4 of NSCP states the following for bending of Fr= 0605, 2 Compression: 1 Ror mmber esting the repress o tou of eymunetry in ' Comprecion on enteme fibers of channels beat about their W21 x 62 steels used as a beam simply supported over x span of 8m. The beam is laterally unsupported over the entire span. F,™= 25) MPa or (45-66) and be justified an not more tha 0.60 Fy.” 160 ig gives the allowable bending stress in MegaPascals in ¢ provisions of the National Structural Code of the rete = 70x 10°C, B 73a D877 Ae : (45-60) 90. Which of the fllowing gives the safe unitormiy dlatabuted Ind that the ny beam can carry in KiloNewton per meter, A 1998 C 1656 Or, when the compression flange is solid and approximately B. 1521 rectangular in cross section and ils area is not less than that of the tension flange: 45-7) a the foregeing: 1 disianes between cross section braced against twist or lateral displacement ‘of the eontpression flange, mi. For cantilevers braced against twist only at the support.1 may conservatively be taken as the actual length, 7 adi of gyrtion ofa section comprising the compression flange pls 1/3 Of the compsion web area, taken about an axis in the plane of the we, Ay aren of the compression flange, ment Ci 1.75 41.05 (Mh F/I? but nok more thik 23, where My i the : Pe Golo ant Mos larger bencng womens ak the end of the braced Tse Jength, taken sbivt the strong axis of the member, and where M,/iM: the Figure AN-A6 ratio of endl moments, is positive when My and Mz have the same sign (reverse eurvatire bending) and negative when they are of apposite signs seructural Eogincerinn 130 November 2002 ‘and Construction Civil Engineering 131 From the foree polygon in Figure (8) sin30*= 1500. y= s000N Ti FEMa= 0} (Tac c0s a)(10)= Ro(7 464) (nc cos 687°}(10) = 3000(7 464) Toc = 2258.39. PEFy=0] Ray+ Tresin 0 Ro sing” Rey = 2000 sin 30° - 2255 39 sin 96.87 Rar= 145976 132 November 2002 ‘Check if compression ses yield From the strain lingram shown: Civil Engineering 133 Reference Vol. 2 ‘DSivation 3 (7109) oF, a Tent 3 7 750000 ae ema | 2 7 300 | aio Ties | 100 Si SMD 176% feat co 1000 ie Tes er 0) TR Tem Bo 0 5.0 ae “Tee | —Frsaaar Tialgoroest poe Fs Me pe Capuinanprina = 20% (790,000) = P158,000 hong iw tb pce gor pay ae © Situation. Fe=9.56MPa (rom Table 31) = 7.91(100) = 7310 MP (From Table3.1) en {= = nem [HOOD _ ben fE han PBI SincelL/ > K, the column is 9 Long Column BOE _ 0.507310) a aka ‘MPa P= FA ~ 4.775(140 x 140) = 93595N P= 93555 kN Situation 5 (43 t0 15) {0 Situation 6 (16 1018) © For slab B; (Cantilever) Reference Vol. 2 135 Situation? aot 2) ‘Acceleration: ote n+ 205 kph = 16.67 m/s = + 20(75)p0= 1.882 myst 8% = 008,0= 4.571" W=Mg=3m0~9 Minimum coefficient of friction when the truck is at rest: a> tan O= 0.08 (0.5) = Wes 0(0.5) inG(D5)-REF(OS) Note: REP= Me 4.57405 sin 4376"] ~1800(1.852)(05) ‘The concrete is a normal weight concrete. Since the steel grade used is hot god 418 Ma, the value shal be auliplied by (04 ff 70. Angle of twist at the free end: L 4000 han = 55 (04 +f 1700) SPS (0.4 + 296/700) Lt o- ) 2 -ter Hay = 318.0020 iG 16 For slab Bs: (Both end continous) T= D*= $ (60)! = 405,000" mm . L 8000 = 1 1000 a Fa = (04%, f TO) = SES + 276/700) 8 apie) M10%PO1aN00 + 200101500) Fina = 227 mam = 800:10°(3000)) For slab 83: (One enc continuous) @=-0.03882 radians x —_ = poe a ‘on irradians Tria (OA # fy f 700) = “I> (OA + 276/700) e roo, = 199 mm * ‘ . . # . * . Structural Ragincering and Construction 136 November 2001, @Siruation § 22 to 24) [Labor cost per hour = 50(1) + 40(1) + 25(6) = P265.00/ hour Tabor Gost per Gubic meter = =U MOE ag gion. am, 0.50 Tour Material cost per cubic meter = 3(150) + 0.25(300) + 1(250) Material cost per cubic meter ~ P775.00/eu. m. Frof overhead, and mibelaneous =90%(688.9 +775) = P4097 per cubic meter P5889 +P775 +P409.17 + P5889 ‘Total cost per cubic meter ~ P1,831.96/cu.m. Situation @5 1027) ‘M=moment in kN-m =. 5/9) = OM =16R/3 > @) = 93.75 KN yr: M=16(08.75)/3 = 500 kNem My= M3 ‘My -250 RN-m Civil Engineering Reference Vol. 2 137 1D Situation 10 (28 10 30) E= 8m =8000:mum; rp= 53.24 mn a ea =1187 SSEIOOCE hen Fis the larger value of since 1 2 = but not more than 06 F,. pee *” [a000(833) /3276] ako 0.6 Fy = 064250) = 150 MPa = 677 MPa ‘Thus, A= 63.7 Mra i ae, Bis ae w= 1656 KN/m Scructural Engineering, Civil Engineering 138 November 2001 naa Coen Reference Vol, 2 139 SeatNe CIVIL ENGINEER Licensure Examination ‘Wednesday, May 4 202 ‘MATHEMATICS, SURVEYING, AND TRANSPORTATION ENG'G. sur NETHUCTION: Seto he corectanewer fr each ofthe following queens. Mark ‘ts anirfor cach tem by shading the bos coresponing tthe tf your che on STRICTLY NO ERASURES ALLOWED, Uso pene n, 2.nly ‘MULTIPLE CHOICE 1. Evaluate tim : | AG e3 B, undefined D, infinity 2. Which the following has the greatest effective sate? Ds 4. A circle of radius 8 em is inscribed in's sector having a central angle of 80°. What is the area of the sector? B 291.84cm D. 475azeme 5. Given a triangle ABC with sides AB = 20 em, BC = 36 em, and AC = 48 em. Find the distance of the point of intersection of perpendicular bisectors to side BC A 18.92cm © 1236em B. 182tem D. 1047em 6, A baseline measures 25 km at elevation 520 m. If the average radius of ‘curvature is 6100 km, compute the sea-level distance. A. 24,996.26, © 24997.97 m B, 24,99863 m. B. 24.995.24m 7. Determine the distance from the curve.x? + yt = I6y + 16r+ 61 Oto point a Fever coves 248 wha Mathematics, Surveying, and Civil Engineering 140 May 2002 ec acu vere 141 A piece of wire having @ total length of 72 cm was cut into two unequal 18 A conical tank 12 fe high and 10 ftacross the top is filled with a liquid that pet. How much work is done in pumping, ‘atthe Segments and bert to from two unexual squares Ii the tla area of the Sajuaes i 180 qc, what she diflerence in the Henge of the’ ovo segments? © 59.475 fielb err © stm B iécm D. 28cm 1», 10. The impact factor of a road with radius 500 m i 015. What maximum velocity can the car travel round the curve. Neglect friction between the road and the wheel. ‘A 97.55 © 8.25 kph B 124 78hph D, 8112 kph 2, 1. A parle moves coring she pare squats se ce der hs ET ste tal ie ata Sais Re aia D, 5.21 m/s? D. 8.018, 12 Determine the aren bounded by the curves y= dr and y= 1) distance between the vertices of the following ellipse? 77a. urls + 16x - 16y - 618 = 0. D. 4.7494, units 107 B B24 D. 23, A thangular piece of land has one side measuring 2 km. The land is to be divide rio two equal areas by a dividing line parallel wo the given side What the length of the dviing ine? © 700 D. 30m B 845s D8 14. The length of the common tangent of a compound curve ts 321.m. Di = 25°, 24, Determine the traffic Mow in a certain highway if the taffic density is 14 1 =267, l= 62". Find the degree of the second curve. Use are basis er Klee with aspace mean seed h Ag cas les/r vohicles/he Br fhe ped on the stadia rod is 1.8 m. ant the line of sight ‘salvage cost of P10,000 a after 10 years. Determine the book value after five years using the double with the horizontal. Find the horizonial distance, in m, ecining balance method am 'A. F38768.00 © Pio.960.00 B. Fas21440 shevop omens tet ; 16. Acylindrical tank open at the top is made of metal sheet having.a total area i | GF BAS square tcters, Wr the beight of Uhe tank is 15 limes its base 25. A spiral easchon curve has a length of 0D m with cena curve having ‘diameter determine the base diameter of the tank. radius of 300 m. Determine the offset distance feom the tangent to AaSm C3m second-quarter point of the spl 525m D. 3.2m A B 677m 17. The volume of a regular tetrahedron is 3182 cubie meter, Find its total C 139m D. 358m surface area. 27 How many diagonastince an undecapn? ‘A. 1247 59. © 16.87 sq.m. AM ce B 1559 sq.m D. W.A2=qm. es 28, Determine the shortest distance fm pots (2) 10 the parabola j= 6: ‘A283 C21 B 354 D. 632 Mathematics, Surveying, Civil ineerins 142 May 2002 mee ee Gearemereaae 143 29. Determine the sum of the numerical coefficients in the expansion of (a + b)*. A 18 256 B 512 D. 1028 ENDS Sine bot the mateeaor and denotinator proich zero, we can apply 2aee? Be Deoey 3x7 slenominator approach 2er0, sec 2r(sec 2x tan 2x)+ sin i = Ssec? 2x tan 2x4 2sine or ‘Again, both the mumerator and clenominator approach 2270, Ene By Gillesania’s Rue ‘Set ¥™ G00] ractian (0,18/s degree) a = fan{2(0:8 /n)}=2sing0.18 /n) =e (@o0n? =29999 0.1280 = 12.89% ozs =12.91% 0127 =127% "ANSWERS, = (n= 5)! ie we Wo BA oan ae ne DA 30 BA BS BA 48 60 Bo me oA DA 58 ‘Ds Actasl or Comected distance at 28 of the earth pe 22S 24188129) «3409 cn aA, (53932) From the right triangle shown (shaded triangle) Has Re 1 = DAO! 188 x= 15.92em me Dy horizontal distance at an 146 May 2002 aw Tn Figure (@): In Figure (): cae Een -(2)- $7459 hod 459-021 m Difference in length = 48- 24= 28cm, 2 ‘Impact ator (centrifugal ratio) = tan = a 1467 p= [36F 16 sl Whent3scc 1° a> J 3609) 916 em 18 mye? dade f= 8-1) Points of intersection: Fre 4x ~ 8-1) r=8r-8 rez yrtB Ow ‘Ty= Ry tan (h/2) ‘Ty ~ 458.37 tan (36°/2) n B PLOT ~ Re tan (62°/2) ‘ R= 286.7 m & Civil Engineering Reference Vol. 2 die Aeaine * Aba * Asis Ammias = $D2+aDH; 4948 = 3D? + nDG. Distance = distance from centroid ‘of volume to point of exit Distance = 412) = 3 m Work 19,603.54 (3) « S5/810.6 fe ‘This is aor annuity problem with A ~ P6,000,00 every thee months or quarterly 80; = 0.0/5 = 0.025 Civil Engineering Reference Vol. 2 151 Ga Slope tan0 =3/2 O63" ‘From the shaded triangle shawn: 080 =5/d an ei) sg Ley 25+ 64/025) = 08 + + 04/68) + 25 (64/625) 5Je= 651.56 divide by 65156 as : ax tax oe 152 May 2002 Mathematics, Surveying, and ‘Transportation Engineering ‘Since the small and big angles are similar Aas «(zy Ag AL fay &-G) == 8485 ka Flown = density = space mean speed Density = 4 vehicles / km ‘Space mean speed = 30 miles/hr x 1.60975 kin/mile Space mean speed = 48.293 km/hr Flow, 4 = (4 vehicles /kn)48.283 km/h) Flow, 4 = 198.17 vehicles/hour H= Dew peLstyr9 = 100(1.8)+ 03, De 103m 1 1803.08.13 Ha 17074 Oifset distance toany point LfromPO, x= =e y= length of spiral =100.m [= distance from 1.8.40. point in spiral [m2 <1 = 50 m (second quarter-point) so oS ‘00)(100) <= 0.684 m An UNDECAGON is a polygon with 11 sides (n= 21) Number of diagonals, D = = Number of diagonals, D = } (11-9) 44 ae te on K yem yrod ye dand x= 2 P= (Q- $+ (= d= 2.828 units To get the sum of numerical coefficients in the expansion of a polynomials, assign a value of one (1) to all the vatiables For (a+ Hp, Sum of numerical coetficients = (1+ 1)8= 256 154 May 2002 Geotechnical Engineering ‘Reference Vol. 2 155 CRESTS Ua aU ee ea ‘Seat No. 7. Which of the following i cloves to the value ofthe factor of safety against crv ENGINEER ray cis Licensure Examination 2002 Wednesday, May 1. B15 D192 8. Which of the following is elosest ts the valle of the factor of safety against overturning HYDRAULICS AND GEOTECHNICAL ENGINEERING SETA $8 oe B28 D.33 Select the correct answer for each of the following questions 9. Which of te following is closest tothe overturning moment acting agalnst arte eles ae ex cere ee thedam in Nem. = oe of your choice on the answer sheet provided. ‘A. 35316 Kem © 285.63kN-m ICTLY NO ERASURES ALLOWED. Use pencil no. 2 only. B s2eazkNem D. 39675 kN-m MULTIPLE CHOICE Situation 2- Water Rows from atank through 160 feet o nc dlameter pi nd then discharges into ai as shown in Figure 20. The how af water ithe i ldcia. Assunen = OLS and nogiet muna asses ofthe following gives the veloclyof flowin the pipe in ps 1. alee the magnitude ofthe total ydosttic ote acting am oh side of te showrn in Figure ‘A. 5487 KN © 8324kN B, 36.52kN D. 41.52kN 2. Two piezometers are installed on a tank as shown in Pigure 27, Determine inthe pipe in feet. the value of C6140 feet Adm © 4125 D. 6370 feet B, 4235 m D, 455M. 3, Water flows at the rate of 350 cfs in a trapezoidal canal having base width of 12 fet und de slope of 210 TV. Determine the ental depth. Assure i 0.015 and S= 04 ‘A 2564 feat C 2512 feet Situation 3 For the pipe system shown in Figure p of the tank in ps 015 forall pipes an B. 2968 feet D. 2124 feat Uheflow a pipe di ef. ips ead 4. A triangular channel with most efficient nai water at the 3B, Which ofthe following is closest to the head lost in pipe Lin feet rite of 4 m/s, Assuming n= 0.018 and 5 = 0.0021 calculate the normal A 74 C456 Septh of flow sn meters. Bsa D. 559 0.763 ma ¢ o6s2m “4. Which of the following is losest to the total head lost in terms of the total Bl osism D. Liam discharge Q, where Qs in cfs 5. An open cylindrical steel tank 24 m in diaineter and 3 m high contains 2: A Oar C osra2g ‘deep of water. Determine the stress in the walls of the tank near the otto the wall thickness is 8 mm. ‘3 09MPa ©. 357MPa 8. 247 MPa D.27aMPa 6A rectangular weir of length m is constructed or cut atthe top , ofa tall tank having a horizontal section 20:m by 20 sm If the Situation 4 ~‘The soilshown in Figure 25 has.a void ratio of0.50 and G=270. i initial head over the weir is 1m compute the Hime required fo discharge 72 =15mie= 3m. cu. mof water. Use Francis formula. 16. M4 the Following, is closest to the effective unit weight of sand in ‘A. 36S2seconds ©. $874 seconds B. 45.35 seconds D. 26:12 seconds cus SITUATIONAL 17, Wc the following is clover ode effi ston a pont Ain KPa ae Soe ec UT ce ren re ura ‘st the upstream side is 6 m. Ne restate uy hich of the following is closest to elie ic grodient of 1 of conaete = 255 EN/ of fction hatin (ick condition teres the base of the dam and the foundation is 06. Hydravitics and 156 May 2002 Geotechnical Engineering A. 088 © us B18? D. 154 Sttuation 5 A vertical retaining wall retins 26-8 deep of soll The sel has a ‘dey unit weight of 110 pef above water table and 120 pet below water tale ‘The ground water thle is 10 feet below the ground surface. The angle of mf the Sa. 35". 19, Which of the following is closest to the total active pressure acting on the wallin pounds perfoot with ‘2. 19005 2, Whi of te flowing gies the total active pressure acting othe wal A. 4500 © 7.200 B 300 . 6,000 25. Whe ofthe following gives he verurning mament about the Wein fb ta The water ble, a depth of 25 ft below the f saturation of the sand abowe 5. Which of the following gives the vertical effective pressure at the midheight Den ivecrd tavern cee A © 4350 B. 2450 ‘D. 6,750 26. Which of the following gives the vertical effective presgue at the midheight of the clay layer after lowering of the water table, in psf. te Sis Civil Engineering 157 Z_ Which of the following gives he vera elfodive presure when Gre Uo ‘water inthe aand layer apo oa a. 60 ©. 5150 B. 750 D460 Situation 8 fn the syrings of Figure 912 the drug had p ~ 900 kg/m and y= 0002 The fw tiavegh the neudls 4 oe Neyled tad le 28. Which of the following gives the slay api Binns A 7528 ae B 6109 D630 28, Which ef the following gives the Reynolds number for the flow in the peed a 1025 ‘ D. 1873 ow following gives the steady force F required to produce the bars Cuan B107N DL S54N 158 May 2002 samo -12, q 160 May 2002 hadron oom pos +25 -m= & 25m mp ae caus HH <2— 124+ a0 24+ 2%)? Be edd By trial and error using the choices, d = 2564 fest = 3804348 Reference Vol. 2 161 ms Water surface area at any time, As = 2020) = 400m Initial head, Fy ‘The drop o discharging 72 m 162, May 2002 DSiteation1 7109) Fe Yel A= 9818)(6% 1) Fi Ryo + We= 9764 188 =S6UEN ME, _ o6cg6t) F5= BEL 25O oa sg SR 76s8 AM = 14; + Wee 3760) + 188(1. 998) =1378.604 Nm RM _ 1378608 b= 137.785 ffs | px 1029 LO" | 10:25(0.013)"(48-78)(03401)" Le "ange ‘HL 1942.33 m = 6370.86 feot ‘Civil Engineering 163 EAH Ex Me Pate cee Hoe hey Be Tyee Tg ry tee P ag7.785)¢ 0+ F +10-ea7n6~ +04 + 19- eee etsy 0050427 FAH) CON ‘Net width = 301.97 mm_ Critical net Area = 301.97(12) = 3624 mm? ‘Netarea = Net with « plate thickness Net area = 322.22(12) = 3866.64 mm Peach -=2624s (0.6(248)] = $39,251. N P=5392 KN Largestcancentrated load = 40 kN ‘Maximum negative bending moment = 25kN-i Point of inflection (Point of zero moment): Distance from right end = 25 4.25» 3.75 m 182 May 2002 Gi Situation 6 (16 1018) R= 296104 Weight of beam = Youc% Arca = 235 x (03 x 05)= 3:525 kN/m 0, = 149.525) = 4835 kN/m R= ifds0 + 4.995(4.5)] ~ 236.104 RN ‘My = 236.104 48) ~4.995(043)*/2 = 101.07 KN-m kN Ne V4 = 236,107 - 4.905(0.43) ~ 233,98 KN. A, 12V*5 pag md sao)” °OM Mae M,N, SE a= 10L07-1m X05) 048 Me= SLAMS kN ne [[me. 4) | bed Ye |[ (37 00m ‘Ve= 135,750 .N = 135.75 KN Van Vet 233.98 2838 = zs27 kN Vy Va = Ver 275.27 = 195.75 = 139.52 kN Reference Vol. = Csituation 7 (1928) Capitalized Cost of the 150-mm pipe: OM , RC-sv t | Gentar RC= FC~Pi,20000000 OM = 88,000.00 + 360,600) = P248,000.00 i _, 268.000 k= Piamow + 2 k= P33a0.60817 Copitalized Cost of 100-mun pipe: xaos OM , RC-S¥ -G+Ir=1 RC= P800,000.00 ‘@i= P136,000.00 + 480,500) = 376,000.00 e=Peonooo + 276.000 , _#00,000-0 O15” 1s015)"—1 Ke FaS69.34433 ‘Annual Cost, AC = Ki~ 3,569344.33(0.15) Annual Cost, AC = P535,401.65 Capitetizesl Costof the 250-mm pipe RC=FC= ‘OM = P280,600000 + 280(500) = 420,000.00. = P1go0,000 + £20000 ., 1,600,000. O15 +015)" —1 K= 4975355 33 ‘The system with seallest capitalized cost she most economical. Thus the 150-n0a pipe it the mast economical ue G2 Situation 6 (22 to 23) For Detail OL, minimum caver is 20 mm For Detail 02, minimum cover is 40 mm For Detail 3, minimum caver is 73 mm 184 Rasy 2002, D Situation 9 2310 27) Patt Network Diagram (Normal Conditions) The paths from Oto 4 are: Path D-F: Duration =3 +5 =8 days Path A-C-E-F: Duration =5 +7 Path 8 E-F. Duration= 54 4+5: ‘Thus, the critical path is Path A-C-E-F ‘Neemal Cost = 1,000-+ 3,000 + 12,000 +3,000 + 1,000 1,000 Normal Cost = P21,000 Part2 ‘Accelerating activities A, C, E,F to its minimum duration: ons By accelerating A-C-E-F, the ertical path would till be the same, Note: Activities Band D may not be accelerated since it not included to the new critical path. ‘Civil Engineering 185 Minimum duration #9 +5 +3+3 = 14 days ‘What if? Ifa nevy critical path is established (say D-F or B-E-F), try to accelerate the activities along this new path Part 3 ‘Maximum available cost is P22-100- The normal cost (fram Part 1) is P21,000. Since the maximum cost isto be 722,100, we may accelerate some activities with an additional cost of 1,100.0, long the critical path: = (Maximum crash time = 5-3 = 2 days) Crash cost “Normal cost Cocos NE" ‘Normal duration ~Acvelerated duration = P250.00 per day 4-3 1day) = P500.00 per day ‘Activity F (Maximum crash time = 5-3 = 2 days) 1,800 1,000, Cost of cashing = 2222-10 pgonon por day Cost of erashing = re 186 May 2002 ST iceman "The best activity 07 activities to crash first are those that have the least cost-of crashing, Among these activities, the best to crash is Activity C ‘while Activity E is the worse For a total of P1100, Activity C ean be crashed for 2 days for a cost of 2 250 = PS00.00, and Activity A can be crashed for 2 days for a cost of 2 £300 = P00 00, For most econoitical project duration, the project may be crashed for a total of 2+ 2 4days. ‘Thus, the job can be accelerated to 21.=4 =17 days © Situation 10 @5 to $0) Direct stress: Rete ® P/d = 250/4 = 62 5kN sens = 25210 «sors nira 35y eeneD T= P(250) = 250 » 10250) T= 62,500,000 N-mm Die + 9) = Se + 200 + 4 E(e-+9) = 250.00 mut Ty. 62,500,000(200) Ro" Saray) 0.000 Roy = $0,000 N = 50 kN > a Te, BSNS) pony Rom Saree) ea. me Rop@ 375 NY ‘Total reaction at D: ¥(@75-625)' = 55901 kN 5496037)? » 16777 kN JO = 54.77 MPa Mathematics, Surveying, and Civil Engineering 18a Howeaniber 2002 "Tmrernion nicning Reference Vol. aan sd UE OE Baa ae 7 Reon reeian agi dhe a Ge op md Op : Gupleerseca ion te ovon Honuaeeerokee teen CIVIL ENGINEER Licensure Examination. = a Se Saturday, November 23, 2002 (08:00 4. m. - 01:00 p.m. D. 31574 10. The losed by the curve x2 + yl = 25 is revolved about the line x 10 0, Find the volume generated. 1c, SURV} DTRANSPORTATIONENG'G, SETA c are D. aie Select the correct answer for each of the fllowing questions i SHC Leh recs Rented PESTER cae icwer foreach item by shading the box comesponsing isthe letter sey aie Gee We enor Myocoiceeonte neon Se ae ay eon CTLY NOE. Use penne, 2 bi ne 12, the distance between points A(2 10, 4) and B(6, 3,2) is 9434, what is the MULTIPLE CHOICE ; wala of? a 2 4. Find the value of xin (3"5)(9%6) = 3°23) BS BS Fes eee St he ea ‘ 13, Determine the radu of the sphere whose equation is aes os - Srpterainm =o 3 4 i 2. Whats the value ofy in the harmonic progression 1, 1/5,1/9, 1/3, 0? 3 cs mo. a vei 14. Whatis the derivative oly ~9°@x)? B12 BB ‘Note: The expression” mans powex 5. 20D older that Ywhile Vis 11% older than. By how much, in percent, a 3G ee is older tan ee ag ce : nan * : 15. Suppose that x ars after founding in 1975, ahs Bom B. 2366 TT rmemberdip of fic) = 100 dsee'+ agen wk what tame aoe 44. PaboUt eto be divide! among Arturo, Berar, and Caloy such that their 1975 and 1989 wa the membership smallest? shares nthe sane ode, orm an arti eugene’ sare A 1983 cay ire unesthat of Arturo How machi Hrd’ sh? ft oe Dies 76,000 : F18,000 steel pipe has its upper end leaning against a wortial Brio DB, P1000 Toma oye round, “Has lovec end ets away cre sectont B= 55° 307, and C= 8° 37 “How fast isthe upper end moving down, in een/s, |is 2m from the wall? oe 5. In the spherical triangle ABC, A = 116" What the value of side c ssa zi* ‘D. 115.65" Ete 6. A cimle is circumscribed about a hexagon. Delermine the area ol etrmine the value of the integral of sin’ x with if hen jonif the area outside the hexagon but inside theclcle 1559 cm = limits from 0 to x/ 6. 1. 738 saci sq.m B. 724g. em D. 74.89g.cm, ih of ene arc ofthe curve whose parametric equations are 7. A lateral edie of the frastums of a rogular pyramid is 1.8 m long, The upper and y= 2= 2090 (ee Ee sdunre Io 1m and the lower base 24 m x24 m square, C14 Betennine the vue ofthe fast n cube meters ae ' Dit ee Cay . nsting from point is given en acceleration which unio os Des increase from sero at Yo 18.m/s? at ind seconds. Te rut veloc a 4, A.golid spherical sto! ball 20 mn in diameter i placed nt tall vertical Alig 4a/s. Find the distance velit the end of & seconds ; iter Gonaining water, causing the water level to nse By 10cm, What is Asm © team ~ the radius of the eylindles? B 165m Diam year ar) cc 1028 B 908 B. 1155 190 November 2002, Taarpertties Eacmering Civil Engineering BD. A contactor can Buy tracks for PSOODN cach, or rent ern for P1200 tuck per days The tcuck has ¢ eatvage value of PIOOOO) at he end of ts: Reference Vol. 2 191 useful life of 5 years. The ennual maintenance cost ee 2. A reversed curve of equal radii connects two porallel tangents 1 Using the annual-cost method and 14% interest rate, determine the wamber z ofc fo ape ol dave pr your tat ech truck mst be used owasrant ts puch, Use eee ee ‘etenine te fa eng of Sinking {and method of depreciation eee ‘ecanir re as ws 1ise Sg ortemepe Posts f student reco following-number of paces after vralking a distance ie WA Uh cg eres ee Se eet. ne bel celine. ‘of 50:m repeatedly as 715, 72.0, 70.0, nd 695, He war aed to newsure the ee alte Eon distance between two points C and D. He recorded tlhe following number ie laws of paces rom C to B or back as 465, 468, 46%, and 460.” What is the distance Interest rate + 10% compounded quarterly fon Coe a Eategtinieeal Soni is tae Zi 28, A pavement has a modulus of elastic B. Pss.992 D. Periz oe 22. At what interest rate, compounded quasterly, will an investment dowtle in oe Detenine Ei teesitae aed oe fan 2 7A hee aie ange from PL tOF.C bya bearing of 6° Band * pereiedormamnael 2B. Tho tangent of a simple curve from PL to F.C. his 8 bear ° Ban a ee theothe angen fom Pw PT. has abeatagol N SS We Ata point 190 Bee ee ees eee a rsa along eee ligt Raesta bei ofl: pent C einrene raring af coieae onihe curve is 62m. radius of the curve. , Continsoss pouring of concrel eran caer D. providing expansion joints and scetional pouring of concrete 3 15i72m D. 176m 24, With the use of an engineer's level, the reading om a roc 80.m away was found to be 282 m. The bubble was levelled though 5 spaces on the level “END tube and the rod reading. increased 49 2884 m, What is the radius of ‘curvature of the level tube if one space om the tube ts 0.6 mm long? A 375m, 335m Bim D. 350m 25. Given the following cross-section nates of an earthwork on an Bem-wide roadway! 670 0 475 +180 140 +150, Determine the area of the cross-section. ‘A, 12208:me ©. Aga mt BL 11837 met DB, 12615 me 26. Given the following, areas bounded by the waterline of a lake and the contours 1, 2.34. and 5. Each contour are at 2miniervah Aq= 4140 2 192 November 2002, Mathematics, Surveying, and ‘Transportation Eupinecring. m3 as Solutions to November 2002 Examination gn 235 = 35 (6 guepgne30 B= x-85 Given the harmonic progression: 1,1/5, 1/9, 1/ ‘Thoir reciprocals 1, 5, 9, x form an arithmetic progression with common, difference of 4. By inspection, x= 13 Xe ay Eq.) Yeinz Substitute Xa ‘Thus, X48 24.3296 older than Z. ‘Let A, By and Che the shares of Arturo, Bemardo, and Caloy, respectively A+B+C=96000 Eq, (1) B=2A 7.2) ‘Since their shares form an. A. P= Common difference, d= B- A =C-B Be AC + Fa, 0) ‘Substitute 8 in (2) to 2): 2@A)=AeC CH3a Eq) Substitute B=2A andl C= 3A to Eq, (1): A+ 2A +34 = 36,000 A= 6000 8 = 2A~ Pi2,000 (Bemnardo’s share) Given angles of spherical triangle: A= 116° 17, B =55° 30, and C= BP 37° 193 for angles Boos C4 sin Bsin C cose =c08 BS" MY cox 80° 37 + sin 55° 20 sin 80° 37" cos a ADIGE i= Area of sector ~ Area of triangle Ne a 2) 25-1) arsine r=5253 em Arouin™ 6% Area of triangle Knee = 6PM Aga = 7.7 99, €m v= 222 76146700] Va 459m ms A n(loys= m R2 (10) Reiissem Solving fox: ‘The curve xt + yt = 25 fs a circle of radius 5 ‘with center at (0, 0) v= (x) x 2xt10) V= 49308 cu. units Volume of ball = (Volume rise)wam Rendered sold Civil Engineering 19. Reference Vol. 2 Ss on Let (J) be the coordinate of the centroid. Then, Abscissa, F = 24t#atee . ~Se2ed 3 z Ondinaie, 7 = LAtEA*VE _ 24602 3 g Prom fa ina Mo ge -3% B= 3 ing) 3™ in @)x3 Wy. gua #9 ing) + 2x) jen function andl equate to zero: 1006-0 + 254) = 0 + 264 =0 When x= 11 fO11) = 109f2(L1)>- a5)? + 264(41)) = 12.100 Mathematics, Surveying, and ‘Transportation Engineering ay When = 4 fa) = 100F2(4y = 4546) + 26 00 ‘Thus, the members ‘that is in 1986, {is smallest (relative minimum) when x= 11 and. ‘The membership is largest (relative maximum) when x = 4.and that is in 1979. From the figure shown, = =? when x= 2m (200cm) ty Theorem: frotocee Let 0-32-40 dem td ‘Limits: when x=0,0=0 Civil Engineering Reference Vol. 2 197 mis EA 2-2sin@; y=2-2omd 1 Note: This curve is a cyeloidl where 2 is the radius of rolling circle and 0 i the 1 angle of rotation (0°= 360° for one are) Length fre, $ = he ty A= (2-205 8) dd dy =2sin0 ad va 7 (2206) a Sm | fe Cater)” ain [ Sind dt | canta so2 [ino 28 eae but fT=cos8 = fF sin one [Fenda Ponte “fot fetal 2. z S16 ‘Note: he length of one are of CYCLOID with paramelzic equations == 1(0 sin ) and y = afl ~ cos @) is Ba. es Mathematics, Surveying, and Civil Engineering 198 November 2002 ‘framsporetion Engines | Reference Vol. 2 ie ov wi | “Annuity Due: 2 Sum ™ (Font +9 Sum = SBS os) = P2.213,657 Difference = 2213 657 -2159.665 = Ps3,992 an ‘Annual cost forthe purchost obthe tuck: emul ct ic Anmulinieest on arvesimert + Arnal operation wal oe sacoance cost Annual depreciation (using sinkang nd) Ace (eoi2 om + CoS (@yt ‘AC (900,000)(0.1) + 20000 + {e20.000 =108.000)018) (olay AC=P2B7 898.48 Let N be the required number of days er year, then: ‘Cost of renting = Annual cost 1.200N = 237,598.48 No» 19824 hrs say 199 hours ou Given: A= P14 000.00 n= 16(4) = 64 f=040/4= 0025 Ordinary Annuity: oom p= HGHOT=M aon 008" Seer Asay dat Ast Ae ‘A= [AC8) + 1467) + 14(478) + 4(0.5)]= 12615 ot Volume by prismoidal formula V= EA, 64m +2Aaa+ Ae) ve Fh +-4(5,010 +4,140) +2(4,650)+3,150] V=36800 cu. m a» :O=as171" In triangle (6) sing & R ringgit = 3 R= 40833 m n(408.29)(9504°) RI Lae eee 140.17 m FortheSom dtnce Average pce SSI2700 5 075 Pace factor= °° 0.7067 meter per pace A for pois Cad Average pce= SAH 10 os pc Distance = Pace factor x Average pace = 0.7067( Distance = 327.91 m_ . Cs (ee ‘Stiffness Factor, k= Vo l= cs E vcparise (0p ms ne ‘Cracks in concrete pavement due to temperature expansions can be Prevented or minimized by providing expansion joinis and sectional pouting of concrete 202. November 2002 conechnical tneineeing SeatNo CIVIL ENGINEER Licensure Examination Saturday, November 23, 2002 ULICS AND GEC {ICAL ENGI iS SETA Select the correct answer for each of the following questions. by shang he box corespontng to the eter DD. Use pencil no, 2ofly of ICTLY NO ERASURES ALLOM MULTIPLE CHOICE 1. A 10-ni long tank contains water at a depth of 2m. Compute the minimum pressure at the bottom of the tank if it accelerates to the right at 2.45 m/s [Assume the sides of the tank are high enough to prevent spillage. Choose the one nearest to the corruct ansvrer. ‘a. 6617 Pa ©. 7.804Po B. 6.486 Pa D, 7391 Pa 2A luce gate wy into a heicrial channel shown i Figure 2. Which of the fol is closest to the flow through the gate per meter width Fallon ca = a Asses tak ta proses ube At Bio be atmospheric and neglect friction losees in the channel ent of contraction C= 0385 and cs of velocity C, = 0.95, Sopa hee Poon c oom Bond D, tos 4. Atha gate in the form of an are of a ciele of radins 6 m is shown in Figure O€ Which is ofthe following 3s chosest to the vertical component of th iydrostate force acting on the gato. a. US'S EN per meter ‘©. 244 por meter B 22KN permet D238 per motor 5. Avtooden block having uniform cross section will oa! in water with 20 em mcg ave th mater ace an pled il (= 9) wi Flack with dem projecting above the oll surface. Determine the total depth ofthe block. Chose the one clocest to the comrectanswe A, 0Stm corm B 132m asa bya oe a 6 A tmsoniy dam (density = sn) i tranglar in cros-seetion with height of 24 m and base width of 12 m. ‘The height of water behind the Werden! face ofthe dams 20m, Deterne the location (measured from the foe) of the vertical reaction acting a the base of the dam. Chose the one lowest tn the conrect answer © 67m D.77m jets from a garden sprinkler are w 2-feet radius as shown in Figue| eplet the velocity ew iy G = O.8Dand celts Do the followii- closest tothe force F appli on each sprinkler uthecene (shown) tomate aiians the following. most neagly give the amount and direction of law? fhe from Lto 2 Casm/hctom sor ie feom 2401 ge Dlg mie from Lto2 following is closest to the velocity of flow inthe pipe? Is © O7lm/s ae 5 D_068 m/s © a2 Di. 135 Situation’ ~ A hydroclectric power generating system is shown in Fi Water Aw’ iteat ar apt seer stem i shown in Figure 13. ‘slower one passing through @ ice eae “The pipe diameter is 250 mm the 3 ee sel and2arei97 mand, respeney pee orca ae i mele persecond most nearly gives the velocty of flow in the pipe in 1A What the oc Which of the following most neatly gives the fictional hed lost inthe pipe D.271 following most nearly gives the pawer lowatts? Neglect minor loses eee ae ae 204 November 2002 ‘Geotechnical Engineering x 10 cis B. 270 D150 Situation 4 - The pump shown in Fh discharges it into 8 nozzle at D. amp 1 150 mand from the pump to the nozzle is 1500 m. ‘The pipe jameters before and after tive pump are 450 mm and 600 mm, respectively, length ‘The atmogpheric pressure i 95 kPa absolute and the vapor pressure is 35 17 =0.82 lorboth pipes, Zs = m. The pump it operate such that the discharge will be the maximum possible 16. Which of the following most neatly Salus 08 Gale Mass = 567 pound mace ‘The solids have specie gravity of 269. After oven drying. the mass of the 19, Which of the fol A 1042 16/60 B07 yee 20, Which ofthe following most nearly givas the porosity of te in-situ soil? A051 co. Boar D032 21. Which of the following most nearly gives the degree of saturation of the in- situ sol? A 73% C 6s 45% DB 35% ane at falar rere determined fo e600 pat and 420 pa, pe Ae lalbekeg ton reat ges we mee otinteal Geir he ca B27 Dia 23, Which ofthe following most nearly gives the angle of the failure plane? Aor C38 B. 5m Ba 24, Which ofthe Following most nearly gives the maxianim principal stress? A 10,000 C9400) B 14,000 psf DB. 18,500 pot Civil Engineering Reference Vol. 2 205 C Ships following most nearly gives the staf following most nearly gives the shear force on each pile? oe C Sep aa . 35 2 Which os ollowing most nny ee the maximum moment in pl D, ‘sums point of conttafesure ata pth af 10 fet Below te eae a 720 bp Co B20 kip D.SokipA Situation 8 ~ mat foundation 100 by 32 ft isto sup por total loa "The formation rats sand wah a densi Lf 0 pe sat ar soos netration resistance (SPT) of ase ofthe mat ‘cclepthof Dy= 8 fect_Allowable setement e inches lowing most nearly gives the overburden pressure al ‘hase the mal foundation in tons per square foot” ea “A 048 02 we abe B12 Assuming a correction factor C, = 0.62, which ofthe following m {gives the allowable bearing capacity ofthe matin tons per squre et ‘A 208 298 90. Which of tho etek ich of tho following, most nearly gives the factor of safety against bean Whe tenn ly gives the factor of safety against bearing er ca Bo 2a Dat In 12. cure = 10.019 097990 os oye 1058 m/s ‘Actual velocity at ©, v, = C, 0: = 0.95(10.154) = 9.6467 m/s Discharge ~ Aa vs ~ (085 » 1)0.6467) = 8.2 mifs per meter 209 13-5 ppp (esespn sv 5 = 0.00203, Considering 1-m length of gate Fy= (Asx) AL Aacee Ariage = HOF 60" « 3? ‘Ag= 3.261 Fy= 9.81(8:261 <1) Fo= 32KN (per meter) H eee irae Fora solid homogeneous body floating ina homogeneous liquid Sty Spat + $F sin ot Vaogeaes = 5" Vinay ‘Let Sy be the ep. gr: of the wood: Tn water: Aq =20) a) SyH=H-20™ > Ea. (0) ‘noi 5 AC -14)= SE (Ax Hy 85 = O8SH-11L9 Eq. 2) [Sett= SoH H- 20 =085H-119 He Stcm= 054m ‘Consicler 1 m length of dam: Ryn= RM-OM. Ry = 8) - FP) Rw Ry = 250014 (12)(2410) Ry 360000 kg= WH 2m Fyayha Fay 1003 20 1) y= 200,000 kg $360,000 x1 = 360,000(8) - 200,000, 32) n=4. nel 1D Situation 7 103) ve Energy equation between ar Cand A noglecting losses: [Eo= Ex) : H Pie, Pe 425 in A ey E a ; Maly Pag 2g Y a Reference Vol. 2 211 + $6 4g. 2a gy o aa +0 ae o+0 4 = SA fis (heoretial velocity) ‘Actual velocity, b= C, b4= 080459) = 78.54 fee Q=CAamn( Q= (x08) 5 s) =O412 F/s Dynamic Force, Fo= 2! » : 015131624) Dynamic Force, Fo force, Fos (7554) = 00.51 Ibs (EMo=0] Al) = Fear = (6031)(2) = 12042 Ibs D Situation 2 (10 10-12) Energy, om 2 AO 1” pg” Sxogt 18 ‘Since E:> Ey the flow is from to 1, ‘Energy equation between 2and 1 Hydrautics and Geotechnical Engineering 212 November 2002, “Assuming laminar flow (Rr ©2000) IE SDNBTVO.OB (Q= 0.00201 m/s Velocity of flow, 0 = 220201 710.08)’ Reynolds Number, R= 222 = as w ‘Reynolds Number, Re = = 135.4 < 2000 (laminar flow, OK) =0711 m/s Since our assumption i correct, then, Discharge, O= 0.00201 m/s = 3600 s/hr = 724 mfr (fxom 240 1) Velocity of flow, 2 = O.7LL mm/s Reynolds Number ~1354 1 Situation 3 13 t0 15) : po 2 ta Vale of low, o= = oa = 2056 mys mil ~ tos7ug' _ Frictional hoad lost y= “Toga Frictional head lost y= 89 m (120) 0.25)" Energy equation between @ and @ (Datum at El 0) HL-HE= Es Power, P= QyHE= 0.15(981)(143.11) = 210.59 kW ‘Since the punp is above the water surface ofthe source tank, the pressure at the inlet (at 8) is alieays negative (vacuum), ‘As the discharge increases, the pressure at B drops. To avoid cavitatios {he absolute presnute a # must not all elow the given vapor pressure of Energy equation between A and B: {using absolute pressure and datum at A) (0= 3.692 mys (velocity of flow in pipe AB “Theeretical discharge, Q = Av= § (045) (.692)= 0.59 ms co: £2 Situation 5 (19 4021) Giver: W=56?Ibs = V=05f We-487Ibs G=209 Density ofthe imsitu soil y= WY — 82 334 byte 214 November 2002 ‘Geotecinica! Eeineering We W=W, _ 567-487 Mowe content nc We, MEI . 8 in 203 ty SAGE, rusa~ 2OABHONES) en 0.7234 fomc=sq 269(0:1643) = 5 (07234) ‘S= 0.611 = 61.1% (degree of saturation) £ 0.7234 Peeks ae ema In triangle = 90" - }= 90° -35.69" = 5631" a e R= atin § = 7571.66 tan 33.69" = 5047.76 71.66 sec 33.69" = 9,100 go 5631" +20= 180%, = GLB Maximum principal stress, = C+R Maximum principal stress-01 = 9,100 +5,047.76 “Maximum principal stress, 03 = 14,147.76 pst 215 ips (hear force in ach pile) Maximum moment at D Mo =H 10 Mp =5(10) = 50 kip-e Axial force at D: Ba+Pat Pe-+ Pp ~200=0 45 + 48.33 «51.67 + 55-200 =0 (OK) 2 Situation (28 to 30) (Overburcen pressure = pressure of the soil removed to place the footing, (Overburdion pressure = y Dj 120(8) = 960Ib/ x (1 twn/ 2000 Ibs) COverburcen pressure = 046 tong/ft? Bearing Capacity from siandard Penetration Test (SPT) Allowable bearing capacity: u~OA1 Non ms (HP) Nae= ouauN N's standard penetration number (the number of blows required to drive the samplor an additional 300 mi) oe tor for overburden pressure oe 2 (Liao and Whitman, 1985) $2, 0'm > 24 kPa (Peck, 1974) Ola ff murden pressure, KPa ‘y= correction factor if the groundwater level is within a depth B blow the hase of the footing Reference Vol. 2 217 Ts 3D, ofthe ground water table phot footing vo footing the dep the roundater vet jon Brom he the depth of groundwater table was not sped cht gr 2 not specified) 508mm o= 99.72 pst < 144 = 4,855.96 pot {1 4,855.96 pss 000 243 tona/sq. ft 5.200 T0082) «= maximum applied foundation stress = 1.825 tons/sq. ft. = 120(8) = 960 psf + 2000 = 048 tons/sq, ft. 243, 218 November 2002 CIVIL ENGINEER Licensure Examination Sunday, November 24, 2002 MULTIPLE CHOICE station 1 Te 500-evton Hock shown in Fgure Oi al rit oF the rough Salton § Tare before eh oct Pa applied alt=) The fee P= 8 2 1 wh etaermg mer da ta a Sas ©. 16 D. 183 Hn wag the distance travelled by the Which of the following, most Sock in meters aie ve seca Bac ering nn yp ron ie nillmeters, i the concentrated load were acting alone? i ca tg mini game A ‘moment were acting alone? © 182 Bost D. 338 : 7 6. Whichof the following most aly gives the maximum deflection, in mum? A326 © 681 B571 D. 486 Reference Vol. 2 219 ee Situation 3 For the sractuns shown in Figure 03? 7. Which of the following best describes the classification of structure (0 sndetermanate tothe secoal degree ¢ leterminate tthe iret degree ing best doseribes the classification ofstructare (3). indeterminate to the frst degree indotorminate t the ascondl degree llowing Dest describes the clasiication of structuge A Sc inde ate th ie dap eS erminaie to the second ¢ Unstable See D. Staclly dtermifiate Situation 4 = The activities. duration, and cost Conditions for a network diagram are shown normal and accelerated accompanying table. Which af the following gives the critical path ofthe network. ‘A BS . Ccper : DB BEF ng is the activity thak is most eas efficient in convert Sorina onesie een ting ty D. 12 If the maximum budget is 22,600.00, which of sives the redcton in the numberof das to comp fc 4 5 B? D6 subjected to a uniform service dead load of 16 KN/m and serves live load of 22 KN/mw resulting in the bending moment diagrem shown, ‘Twente Percent of the ive load wall be sustained tm nature, while 80% will be Applied only intermittently. The concrete steengih f= 172 SMa, The iiodulus of elasticity of concrete is given by the expression Ere Sri) and the modulus ef rapture ts ven by the expression f= 07 Structural Engineering Civil Engineering 220 November 2002 and Conseroction Reference Vol. 2 221 Gf, te wan provisions fhe TOP Natal Suc Cos of TS. Ifthe itantancous deflection due ta the combined serie dead ond Te uber load is 5 mm, which of the following most nearly gives the additionel Fo.suo Unless sfinoss values are obtained by a more comprehensive defection (i aditon tothe inital deflection) were® yrs, ander te ‘analysis, immediate deflection shall be computed with the effective ‘modulus of inertia a 612mm D. 735 mm: ‘a secondary member inclined at an angle ‘each 250 min wide and 50mm thick The thick main member, It x. The load P= ied ce given in Table ho 1088 sleflestion is comput Are given in Table OL ‘of gross concrete section about centrdidall axis, le ines provides thatthe allowable load reement i 8 10 the grain between 0 to racked section transformed to concrete th Formula where Mg cracking moment = ‘My= maximum moment Ia ye distance from centroidal axis of gross. section, neglecting, Px ice eee ie reinforcement, to exreme ibe in ts aa ray be taken whore P isthe allowable load parallel to the grain end (isthe allowal - ea] positive Thad perpen he gan Rete Se aa tnd negative moment sections, 16, Which of te following most any gives the minimum required capacity of less valies are obtained by’ 4 more comprehensive analysis, 753 EN C487 EN longterm deflection resulting from creep and shvinkage of B B20 IN, Beast embers shall be determined ‘by muliplying. the mediate 17. iffy wll accr inthe secondary member, Wich ofthe following most Sefton cause by the sustained ld considered by the factor ‘early gives the minimum diameter te bol? ™ © ism regen Eq. (10) ‘6mm, D. 20 mm aes + 50p i ne will occur in the main member, whieh of Tc reinorcement rato for non-presiressed compression reinforcement ee ee ees aa Brnldepen for simple end Continuous spars, amt at suppor for C 1mm “antlevers = ‘tmedepen sustained onde to be eal te timber members, each 40mm wide and 7S mm thick are joined The diameer Syrana oles is 2mm larger than the diamoter of the Blt The ter ia itress grade yakal. The allowable stress ructural timber of loads on one bolt 3 months ‘No adjustment is required for Toad, duration, moiture,temperatu : ¢ following most near ive ive moment of inertia at by Semmipernl up action. w Sesappes Gmcinasinezairemene 18, Whole lloring net nea pel et ares of he ber mec ‘a. Ob7s8 ma C. 0.005784 24,000 min? © 15500mme B. 0.00686 mr D, 00815 ana an whee D.37400mme 14. Which of the following most nearly gives the effective moment of inertia for \ While following most nearly gives the maximum load such that the the continuous member, allowable tensile stress on the member is not exceeded. A. 0.00686 m4 C. 0.00815 m4 A. 690 kN C 412kN B, 090879 m™4 D. 0.00703 ma B SEER: D. 375KN Structural Engineering 222 November 2002 ‘and Construction ee, Zl. Whech of the following most nearly gives the maximum load such that the allowable load on the bolts is nat exceeded, ‘A Osi © orkN B 15k D. 390KN ‘Situation 8 ~ A Bill of quantities is to be prepared forthe square isolated reinforced ‘Gone colum and foot chownan Figure 08. The swell factor for the backfll i 22. Which of the following most nearly gives the quantity of excavation, in ceubje meters: ‘A 0218 © 0465 B asa D. 0570 23. Which of the following most nearly gives the quantity of concrete in cubic meters AL 0568 © 0200 B 0372 D, aais 24, Which of the following: most nearly gives the quaniiiy of formorark, in square meters: : ‘A 302 4524 B 256 D. 580 Situation 9~ A rectangilar column is shorrn in Figur 09. Its seted upon by the following loadse 25, Which of the following most nea sillimeters ‘5. 300 26, Whicket the folowing mot natty gives ta Which most nea ea ofthe minimum required steel reinforcernent i tallimeters! eS A. 5250, C. 3820 i, 6a D. 4100 7. Which of the following most nearly gives the minimum diameter of the reinforcement, in millimeters. A cas B36 D. 32 | eae ae ee 18 Rae eT (oP xm OAL Figure 09 Civil Enginee Referemse Vo 2 227 ‘TABLE ML: Allowable Loads (KN) on One Bolt (Double Shea) Normal Duration | auats) =| F G Sab ry eEGS Bee uuuse owus: i i Beas i 2m W06-n 2am kar tree: él eeHEs Bubeec WUNESS BNeaS BESEs MEEES uNEe ‘AT MpseaN Reference Vol. 2 229 5 Teor e as ie i 8 x = a 58 te = . Wien i pallgpore 5 iB eee. Ghesstamsw 9. awe - (f= 0.2N = 0.2(300 + 408) f= 60+ 8t (EFH =O aera FA 520 gages 089 stowage mga 9694 Integrate both side: Srase-ovei eon] 5 = 0.334(6"- 2252) - osngs: -2.2522) ~ 066565 -2252) 5-16.56 m © situation 2 (4 t0 6) oo | i. Deflection due to concentrated loud, = Mi’ 466N—m? y= [4220/8] x 229 = Jee S100) 233, mm (200,;900)(100,000,000) See SI) ae ee & > [7 000(100.000,000) ‘Maximiam deflection, 8 = 8, + 8 = 2.38-+ 3375 = 5.705 mm nal hinge gives ane equation (zero moment) Figure (a) Number of reactions:3.+141=5 Number of equation =3 Thus, the structure is indeterminate to the (5 = 3= 2) sscond degree, Figure (®) ‘Number of reactions = 3436 ‘Number of equations =3 +25 ‘Thus, the structure is indeterminate tothe (5 5~ 1) fics degree. Prgure (9: CNuieratrarsiaire 2414285 Number of equation 342 3 ‘Ths he structure otaically determinate © Situation 4 (10 to 12) Pay Past Network Diagram (Normal Conditions) 1,000 + 9,000 + 12,000 + 3,000- 1,000 + 1,000, ‘Nomnal Cost = P2i,000 232, November 2002 ad Caceres Tames astm available cont is P22.00: ‘The nos cost (from Part 1) is P21,000. Since tht maximum cost is to be P2200), beac eeny olan ecce acirdiag erin x dtHloaal ont of 160000 ‘Accelerate dtems along the critical path: ‘A: (Maximum crash time = 5= 9 2 days) Crash cost—Normal cost ‘Nonmal duration = Acceleraisd duration 4St0 1008 720000 per day Cost of crashing = Cost of crashing = Activity eee 7-5 2days) Costoferthing = 25°0=3009 = posoq pe day Jnsin crash ten = 4-3 = 1/dey) 150910 ~ 750000 per day hing ~ ‘The most cost effective activities to crash first are those that have the least cost of crashing. Among these activities, the mest cost efifcient to crash is e000, and Acuity can be cashed or P4000, Thus the total cost of crashing is 100000) For most economical project duration, the project may be erashed for a total of 2+ 241» 5 days. Thus, the reduction in the number of days to complete the project is 5 days, Situation 5 (13 1015) Civil Engineering Reference Vol. 2 233 —— Myo fle vi 4-07) PF, = 07 472 = 2.903 MPa igs OOU715 mt (as given) Y= 810mm 2.903(0.007153 10004 i 65 956,290 Nm ~ 66.955 KN 1, = 0.00573 m 6.956)" i = (S Jom + [.-( $2) | onsm = 0400878 mt For the continous members - Hauspornamant *(!, mune moment j~ (Waconsee daar 59527 eet: mma eae monet pp) Solving for I at maxirwam postive moment at midspan) 2.903(0.0138% 1000") f= SMU Os nm Mer ‘0-198 94,040,845 N- My =S808 ke Me= 145 kN ke Get) eos «| || ] coouszay k= 000785 0.00578:+0,00733 2 ‘Additional ong term doflection = long term deflection i, BS = 0.00686 me {since there is no compression reinforcement at mids (after 5 years) eee ‘Solving for the long term deflection under sustained Yoading: ‘Instantaneous deflection = 5 mm (given) ‘Instantaneous loading = 16KN/e + 32 N/m = 48 KN/m ‘structural Engineering and Construction Sustained loading = 16+ 308 (3) = 204 KN/m ‘Since deflection is directly proportional to the load: Braga 5. ma" a8 Saag = 2588 mam ‘Additional long term deflection = 2.333% 2 = 4.67 mm Situation 6 (16 t018) “Two S0-nen tk Part Net area = Net width Thickness Not area = (400 = 27%3)(75) = 23,525 mmt Pan For faire in the main member: on parallel to grin Se na, P = Allow tension| to grin x Net area Load in each bolt at an angle to grain = 44/6 733 kN Allowable tension parallel to. grain = 24.5 MPa (Table 02) 0 30° ‘Angle to the grain, ‘P= 245(23/925) = 596,163 N - 596.163 kN 236 November 2002 Sioa Coneeraclon art 3: (Yakal belongs to Species Group I) Bolt diameter = 25 min amember joint (single shear) of equal thickness. For this If the tabulated load for a piece twice the thickness of § eee re 1D Situation 8 (22 to 24) do. = O01F Quantity of exc V = O(063((0.1 + 0.25-+ 0.6) = 0.342 m? Quantity of concrete: Footing » 0.6(0:6)0.25) = 0.09 mm oe anes a aE te ake ee aa ‘Column = 0.15(0.2)(06 + 2.05) = 0.1095 ms ee ae ‘Total quantity = 0.08 + 0.1085 = 0:1995 m? From the interaction diagram, py 0 Quantity of formwork: A agram, pp 0017 Pooling = none apn Se Column = (0.15x2 + 0.2+2)(0.6 + 3.06) = 2.56 m? € ‘Total quality = 256 mt ‘Ae = 0.017(600 x 400) = 4,080 mut The miniusm sumber of bars fora tied column is 12 Situation 9 (25 to 27) Solving for the bar diameter with four bars: P= LA(Q50) + 1:7(500) = 1480 KN Ave §Dix4=4080 D=s6mm My , 2225 canny,» Me = SO Eccentricity, ¢ = 0.3 m= 300 mm Civil Engineering Reference Vol. 2 239 ‘Seat Now CIVIL ENGINEER Licensure Examination Wednesday, May 14, 2003 SETA INSTRUCTION: Select the corract answer for each of the following questions Mark only one answer for each item by shading the box corresponding tothe letter ‘of your choice on the answer sheet provided. STRICTLY NO ERASURES ALLOWED. Use pencil no. 2 only ‘MULTIPLE CHOICE 1. If log, 122.262, find the value of log , 3 A075 «Cos 5. 025 D.1o 2 What is the sum of the numerical coefficient of the expansion of (4+ b)&? A 6h iS BS D. 3. aand bare positive numbers. a, ~4, b... forma geometric progression. 14, a, b.form an arithmetic progression. Find the value of a AG fear D7 value of x in the equation (x 59) (1-2 =7 i a ag @ 0 i) Dz i. tla ae 8 cm, 24 can and 34cm, respecte, Find the e median tothe 2i-cm side, inem. = CBs 3 D208 6 having an area of 22439. m. is inscribed in an octagon. Find the area of the octagon. ‘A, 28.659, m. ©. 236559 m B. 24525q m. D, 2467 sq. m. 7. Find the radius of the spherical wecige whose volume is 12 cu. m. with @ central angie of 18 rar, D. 215m vertex is the opposite face of a tetrahedron if an edge is 50 B, 40.825 cm B, H2i4cm 9 A truncated prism has a horizontal triangular base ABC, AB = 10 cm, BC = ‘Vemand CA = Sem. The vertical edges through A, B, and C are 20 cin, 12 m,n om long espestiely. Determine the volume ofthe pris, nc civil 240 May 2003 “Transportation Enginecring Sn aa% TO”, wsing Pappas Theorem determine he vole pencated by eval —— 23. A structure employed to direct the flow of current in rivers with a view to the area in the first and second quactrants bounded by the ellipse 4? + 25y the establishanent of mors lavorable Hand the Hoss about the Ras Reronlehne of more favorable and fined chanel and to proven scout D. 378 a oy © mole u tio of the length af the minor axis to the length of the major axis 24. Giverta tide slope of 21, youd widih of thee sectional Toye at = na of 21, a road wi ‘mand a cross. area o se TOR TO 31:74: m, nd he valet yin tetallowing comrsccion waa 062 Doss 98 o 74 12. Whats the radius of the citcle t+ yt + der = 10y-92 = 02 +2 x az AB cu A 1293m © 451m 33 D.10 B+27im D. 426i 18, Points C 5,7,2) and D (4,1, 6)are7.28 cm apart. Find the value of. 25. A car driver travelling at a speed of 65 mph approached Sis © fan travelled 72.2 m during the peiteption reaction whe What res heat B gen Dian 2 PIEY (perception, identification, emotion and valition) timein seconds? M, What ithe engthof he curwe =A 23 : C25 ‘A Br 2 Bn Di ar 18, What s the derivative with respect to xof 2cos (22377 farang) c +x) B. -6rtsin@ D. +33) A S35°E © same 16, Whatis the curvature of the curve = 16x; (4, 8? B. S.45°B D. S40°E ‘A008 © “0066 27. The following interior angles of a triangular travesse weve measured with i ass fatto eae thesane rion Whats a most pees vluc othe Ando? 17, Using limits 0 to »/4, find the integral of siné 2x dx. Jalue (Degrees No. of Measrem. A O37 C0322 poh cen) py ate oot os: wel ie = 8 yar salage value» 10, é a 3 18. Cost of machine ~ 140,000 pesos; useful life = B years: salvage value = 10, 3 (000 pesos. Determine the 4 year depreciation using the double-declining A. 4935 cos balance method, in pesos. Bs AOE: D. 40.77 “A 14765 © 16182 28. What is the central angle in degrees ofthe curve whose radius is 200 m and B. 7312 D. 15634 the distance of the midpoint of the curve to the P.. is 14.20? 19, Terms of sales: P60, 000 payable in 60 days or pryable in 15 days, AB ca Find the equivalent annwal rate of simple interest if paid in 6ft days, os Dat ‘A. 225% © 247 29. The horizontal axis of a transit was inclined at with Ue horizontal due to B, 343% D.220% non-adjustment, The first sight had! a vertical angle of SO" the neet had GOS 20, Find the regent value in peace of an any of 20000 pesos payable Determine the error in the measured horizontal angle annually for 8 years, with the first payment at the end of 10 years, i money AT ae C was" Ss worth 5%. B 792 D892" ‘A. P70.354 P77A51 a ighing 1000 kg rans at 60 ky inctlar curve END“ is of 100 m. What force: ld there be to svent the car from siding? ih 2450 N B BEN 2. A compound carve has a common tangent 520 m long, The first curve passing through the PC. is a with a central angle of 30°, Find the radius ofthe second curve angle is 35° Solving for 0 in triangle ABC: 1SF= 38 +24 20424) co 0 0» 360/245" £08 (0/2) = F/R fa 4bnformaGP: : 7 a (Common ratio ae ce Aecugea 8 x Avie ‘Aasngen = 8 508-1397) sin 5? Agen 236.27 108 spherical wedge of central angle 0 i6 directly te Sekine of sphere, which is a spherical wedge of an -Landa= 48 Note: Two complex numbers are equal if their corresponding, real and imaginary parts ercequal, Le. e+ bi =c+ deifa= candid Ap= 39,686 cm? 2212 ceratc ; an Volume = 39.686: mio lipes: 42+ y= 100 Bohay shee Tin own cee AG 4 Volume 3 nate= 3 sate eee ; Tebl wep tpetoeeh aD x(4)= de Volume = 4 n(5)(2)'= 88.776 cubic units 3 [No nce to use Pappus Theorem as suggested im the problem ou Sat oy A= 0 Suir loe 1H Divide both sides by 144 2 “1 4 a : Zoos (2+) = set sin +9 Mapeien a8 ine ea tate of miro tojr sien $= 07 Mathematics, Surveying, and 246 May 2003, ‘Transportation Engineering Die fovm« tet aees ana when x= 0;0=0 when x= 5/4; 0 = n/2 [fetes : Provo “4 footow thx 0.285 dm BY-BYs z aver Fe(1-2)" esv xe wy.=t4o00( 1-2) = pasos a vy =140p00(1-2) = psopseso dan 759,062.50 -744,296075 = 184,765.605 Civil engineering Reference Vol. 2 247 rom the cash flow diagram showns ‘This is a deferred annuity consisting of 8 annual payment of F20,000 with the first payment to start after 10 years. 248 May 2008 249 On aes ay esa sn IE di Sot ‘p= speed = 65 mai/lir = (5280 f/m). (1:m/3.28 t) x (1 a m) #210004 kn/ he" 298s ee es S=722 = Se AE 48 see time, t= = = sires ~2AS seconds ‘Bearing of ABN 25° E Azimuth of AB = 205° = 360"/6 = 60° 25 ‘Azimuth of CD: = 25" +20. Dike Jetty = a wall or other barrier built out into a body of water to shelter a ‘Sum of interior angles harbor, protecta shoreline from erosion, or water currents Error = 181°-180° =1° Levee «1. a hatural embankmnent alongside a river, formed by sediment during times of flooding. Dan arial embariament alongside a river, built to prevent flooding of the surrounding land. Mae engtracton tunneling machine: 4 machine designed fr boring eae ‘through hard materials such as rock Hy eae a6 oli & ib oO 346154 x60 11.5385" ‘Sum = 26/30 = 41? 15 8401" = 40765" Radius, R= 200m ‘Total = 60’ (OK) sca aid A As MEV of angle A B17 Hed) HSI 48) s+ 24) -4H2) ‘The distance from the midpoint of the curve to PLL is the extemal istance. It may also be referred ts the shri liana fom the care fo 250 May 2003, Error in horizontal angle when the axis ofthe telescope isnot horizontal Error, £ =e tan Where: # = ade of inclinalion ofthe eleoeope sae, wally fn mine oe cena retin ingle fact EE error in horizontal angle in minutes or seconds When two observations are made, the total error may be expressed as: Error =¢ (tan 61 - tan 03) Where: 0,» first vertical angle = 50° Oj second vertical angle = 30" Error=4.[tan50°~ tan (-20°)]= 7.076415" Exror= 7458" Civil Engineering Reference Vol. 2 249 ————— as Distancia during he IBV kine, = = 65 mi/ hr S280 f/m) x (1 m/3.28 ft) (Lm / 1000 Bla weahinn ct ee 8 time, (= 5 = 22-248 seconds 26 ¢ Bearing of AB = N25*E Azimuth of AB = 205° = 360"/6= 60" Azimuth of CD: * = 205+ Ba ra mas ‘MPV of angle A =41° - 13.8461’ = 40.769" Redius, R = 200m, The distance from the midpoint of the curve to PJ is the extemal distance. Itmay also be refered to asthe shore! ate frm he curce to Civil Engineering Reference Vol.2 251 CIVIL ENGINEER Licensure Examination y May 14, 2003, Errer in horizontal angle when the axis of the telescope is not horizontal Error, E=etan0 Where: SITUATIONAL = angle of inclination of the telescope axis, usualy in minuiss or et atl : Situation 14 ck of wed 606 mk mets in dimension was own = abwerved vertical nigh ofthe object : into the water and floats will 0.18 m projecting above the water sesione Eemerd ice Sera ecnd “The same block was thrown into a container oF quid having a specie is with 0.14 m projecting above the surface. ‘When two observations are made, the total error may be expressed as: Seats 2 San 2 Determine the specific gravity ofthe block. Ervor = (tan 0 ~ tan) stein the specific gr oct ae B 0583 D.O751 ertical angle = 50° 3. Deterige he weight ofthe Blk ‘vertical angle = 30° a r ae B 1509 kN D; 0851 kN Be eet (Ze Situation 2 A trapezoidal channel has bottom width of 6 m and side slopes of 2 ne orion 1 wert Wf the depth of flw is 12 m andthe flow e200 n)/see, 4. Compute the specific eno fe ee aaa C1 Mm B. 254m D.102m 5. Compute the slope ofthe chanel if = 0.014. ‘A 0.00023 C. ogoo7s B. o0oot7 D. agooss 6. Compute the average shearing stress atthe boundary. Acre C07 Pa B55Pa D. 82P 6, Situation = From Figure O01 tis shown thatthe gat is 1.0m wide and is hinged ‘at the bottom of the gate. a =e 7. Compute tw hydrostaie fore in kN acing on the gate A. 2640 C1230 B 1830 D. 19.60 8. Compute the location of the center of pressure of the gate from the hinge, A. 0.667 m. C0517 m B 078m D. 0333 m Myaraulles ana 252, May 2003 Geotechaleal Ragineesiag Determine the minimum volume of concrete (unit weight = 23.6 KN/imw) needed to Keep the gate in closed position. A. O62 me Co. Sitoation4 A pir # Gauaw of pti jtaation 4 - polwork consuls connected 10 Pi ns 2nd 3, whe Single pipeline dup to point D. Pj eer peleee ach are partial ea Ato B i 10 Mers/ace and atsuming [= O42 following: re Paes ee ee | | “2200 30 En 3,200, 200, 4 2300, 0 10, Rate of flow in pipe’ in liters/sec Rowe ee Cis % 231 Dam 11. Rate af flow in pipe? in liters/see. ‘A 7.68 © 956 B93 D638 12, Total head loss from A to D. ‘A 168m © 212m %1bm D, 288m jon 5 = A tank of carbon tetahlride (CCU) has a mas of SOD ae « jume of 0.315, 13, Whats the weight of carbon tetrachloride in KN? © 580 D. 540 4 © 1691 D587 15, What is the unit weight in KN/in?? pe ee B. 1557 Dae Situation 6 - The table below shows the laboratory results of the sieve anal ‘of asamp, Plot the grain size curve il in the attached Graph 11-1 16, Determine the nearest value to the elfeetive size. 4. 045 B00 5 17, Determine the nearest value to the coefficient, of uniformity, Co. AB C10 Bo = i sein oo ee i ec 253 ©. Ge ipoeny graded grave 5. Sw fell peed sak 60 0. T 100" 0.149 10 20 07a 4 Situation 7 - The following data was obtained from laboratory tests for a cohesive specimen: moisture,content, w, was 225%; G, = 2.60, and to determine the approximate unitweight, a sample having a mass of 2240 g was placed in a 500 cin? container with 382 cin! of water required to fill the is the nearest value to the Total Unit Weight of the soil sample in KN /an? © 1862 BD. 1971 nearest value to the Dry Unit Weight of the soil sample in cae B 161 D152 21, What is the void ratio e? ‘A. 0.678 C. 0.866 8, 0582 D. 0784 Situation 8 = The ground water level ma thick, very fine sand depesits located 20m below the ground surface. Above the free ground wate line the sand seogute bycapllry sen The unt woh ete satan 33 a 2 What is the total stress in kPa on a horizontal plane A located 45 m below the ground surface? AO7562 © 8236 B. 6873 D. 9135 3, What is the pore water pressure in kPa at this plane? A815 C1: B. 2453 D. 4182 A, Whats the effective vertical stress in kPa in plane A? ‘A 7212 C. 6083, B 5a63 D. 8736 tes ihe follwing pope: woul mo = 040 capillary constant (= 0.20 cm’. Free grou ‘water level is 80 m below the ground surface. be ere 254 May 2003 Hydraulics ana ‘Geotechaleal Engineering, Bs 2%, Find the height of capillary rise thesilt. Capillary rise s given ash= C/ (ex D7m Fal he wert eflective sess in kPa at 5m depth, Assume uni weght af solids = 265 KN/m? and that the soil above the eapillary action rise and ground surface & partially saturated at 50%. ‘A 176 C158 8, a D, 134 Find the vertical effective stress at 10m depth. Assume unit weight of solids ~ 265 KN/m! and that the soil above the capillary action rise and ‘ground suroce is partially saturated at 50%, As 105 Cio DMI Situation 10 - The following data were obtained from the Atterberg Limits test for a soil ‘ 2. 2, 30, Situation 11 = A footing 6 m Liquid Limit= 41.0 % Plastic Limit= 21.1 % What is the plasticity index of the soil? 15% 19.90% B 25.52% D. 054% Ifthe in situ moisture content of the soil is 30%, what is the liquidity index of the soil? A 094 045 B. 0.60 B. -084 What would be the nature of the soil? A dense B. brittle solid 10000 LN. The base of te shear site (Slven le Tecaghtsulinate bontng capacity for uae footings. qu 6 13 cH +1) Ne* OOD Ny Determine the gross fou pressure in kPa. ie C5 Leen B28 termine the net foundation pressure, A 21 Ge B. 387 D. 183 ‘ Calculate the factor of safety of the foundation agetnst complete shear failure under the undrained condition (both gross aid net), Side colesion an the foundation may be neglected. 1 ES CFS pon = 63 FSna= 103 FSna* 75 B, FS 53 D. Fp = 78 FSmc= 8 ‘Note: Darcy's equation > = ki and q 0A; fon, fon, bia... kay are the coefficients of lity. of indivic Se ena teak ig etn BA. Derive the ex for the equivalent coefficient ty Derive the expression For the equivalent coefficient af permeability in the Percent Pavsing, % hid ec andy = 210% ka =1 105, sec, determine the equivalent coeffi yn he horzontal direction, nn Tur auertcoetiient of © 7257x104 B. 8,05 104 D, 50061 x 102 70; determin the total ow qin omtfanc. S71 0674 a Particle Diameter, mm 256 May 2003 Sroteehnieal Engineering UNIFIED CLASSIFICATION SYSTEM (USCS-114 ot Teka Nes ae. re om 2 ee © © Ses rena Bh sfe|ele}elal« 258 May 2003 D Situation 1 (03) In Water peaia Smeal Saver Swat dt=h- 0.18 > Eq. (1) In another Liquid Draft = Seat, Siig i anotrer ex (6 = 0.9) f= 054m > height of the block Substitute tte Eq. (1) Soon 58) = 054 - 018 > Specific gravity of wood Weight of block = Yoo Vite Weight of block ~ (81 « Oc67)[(06 x 0.6N054)) ‘Weight of block = 1.272 kN Sona" 0667 @ Situation 2 (410.6) 2 ‘Specific energy, H= 2 +4 = Pe age ‘Specific energy, H = 1.409 m 5 = 9.81(0.6868)(0.00034) éitasnisee From the FBD of the concrete block (Fy~ 9) T+BFaW BE = teVicwe= 9.81 Vou = Yeo Vere 23.5 Vere 5.2824 981 Voom * 23.6 Veen Voone = 0.3796 2) 260 May 2003 Geotechnical Engineering ates 261 ———— DSinationa wo Sinton 55%6 15) Weight: Weight, H's wag = 5000.81) Weight, Y¥'= 4,905 32 = 4.905 ken omi5 p= 1,587: kg/m tiy- Soepee | y= 2eco.azy2000}0.00"_ A 1.549 m, az > Ea. (1) in regulating, flow mechanical behavior The Average Gri Size diameter of the soil is Dep 262, May 2003 geek —— Pait2: Coeificient of uniformity: Percent Passing, % Bes 8 8s 8 8 on You Particle Diameter, mm 05 264 May 2003 © Situation 7 (29 to 21) MC= 225% G= 260 w- 4) Sues On- PI Hydzaulles and Geotechnical Enginecring Since G,> 6, Cs between 1&3, and mow than 50% pass No. Sieve, the soil is SW (Well graded sand) rd] 18.62= Bo22H022) oa ©0678 @ Gi 26 (eg “ 4981= 152 kN bee ed eae 152 kN/ae {D Situation 8 (2210 24) Total stress in A: pr 20325)'*203(0) pra o13s Kea Pore water pressure in =9.81(25) PODS ka Eifectve stress in tree peepd Pepin pe Feabias 2a ve e605 ka 1 Situation (25t027) Note: ‘water, The water is pas ‘wih depth below the water table ‘Below the water table the soil is saturated and the water is called free itive (compression) and increases’ ‘Above the wate able ap to the, ight of copiiary nee i the | i id the, water ‘wate. er eee eee 7d ated cesar wal ‘with height above the water alo sone, | Civil Engineering Reference Vol. 2 265 ‘Capiliany Hse Jono tex Sn shen || a0 is ai Bapeeoa re sa | E | natste lee me aab ee? at fn | atm § bare we 1" +04 x981=2 me a 1,729 KN / 2701+ (05)(04) 1404 9.81 = 2033 KN/im? pre 21729(2) + 20.398) = 104.448 10a ‘Pore water stross at A: Po -9.81(8)=- 29.43 kPa Ure perp] 10.88 = pe +(29.3) pe= 13386 kPa See eo Cur ae 4 <0 => Semisolid alate ~ high slang, Drie (sudden) Fee a upen tea ett mae or tenis ce ee re ‘viscous fluid Since LI = 0.447, i. 0 LI <1, thenature of the soil is PLASTIC 2 Situation 11 (31 to 33) sono as Ge 1S chet rN 478M, Factor Sty, gpa Se = 2 rate Factor of Safety, FS, Pactor of Safty, Fi = — + = 216166 23 2 Gem Gaceienrronn go Oo 1 (6) Gross foundation pressure, p= 277.8 kPa 268 May 2003. ‘Considering 1 em width: A= Hx 1 (12000) ‘A= 1.200 cme 47 = (8025 104 JO070)(1,200) 0.674 cm/s per cm width Civil Engineering Reference Vol. 2 CIVIL ENGINEER Licensure Examination ‘Thursday, May 15,2003 STRUCTURAL ENGINEERING & CONSTRUCTION INSTRUCTION: Select the correct answer for each of the following questions. Mark only one answer for each item by shading the box correspo of your choice on the ansiver sheet provided. STRICTLY NO ERASURES ALLOWED, Use pencil no. 2ony. NOTE: A times B is represented! by A= B or A'B, A raised to the power mis ‘expresséd as A". All problems are 3.33 points each, SITUATIONAL . Situation 1 - The concentrated load shown in Figure ME-S8 acts at point A, 1. If the concentrated load acting at A i replaced by an equivalent force- euple acting at point 8, whith of the fllowing mast nearly gives the ‘magnitude of the equivalent couple, in kiloNewton- meters: a2 Me te BB D2 ‘Ifthe concentnated load acting at A is replaced by two horizontal forces, one fcting at point B and the other acting at point C, which of the fallowing ‘most nearly gives the magnitude ofthe force at B, in kiloNewtons: ‘A. 450 60 B. 500 D. 550 ‘Which of the following most nearly gives tive magnitude ofthe force at C, in kiloNewtons ‘A. 550 ©. 600 B. 450 Situation 2 A plane joaced as shown in Figure MIE17_ 4 Which of the fo most nearly gives the reaction at G, in Newtons: ‘A. 810 C78 B. 645 D920 5. Which of the following most nearly gives the bar force Aif, in Newtons ‘A. 810 © 65 B, 1900 D. 2678 6, Which of the following most nearly gives the bar force [D, in Newtons ‘A760 c DB 40 5 by Situation $ ~ A. beam has a Tsection built up by two plates as follows: a 200 millimeter « 40 millimeter plate as flange and a 100 millimeter x 20 nullimeterplaisas web. The maimum shear frce acting on he bam is 60 loNewions. 270 May 2003 7. Which of the following matt nearly gives the moment oF iarta of the Situation 4 ~The beam in Figure AN-2 is acted w Situation 5 ~ A Seructural Engineering ‘and Construction section about tha neutral axis, in mullimeterst. C. 8,700,000 . 24,500,000 8 following most nearly gives the shear stress at the neutral axis, © 194 D. 271 8, ywing most nearly gives the shear stress on the web at the junction of the flange and the web, in MegaPascals: A196 Cus B. 43> D. saz by a uniform load asshown. ‘product £1 = 1,000,000 where the modulus of elasticity of the material (©) 5 in Pascals, and the moment of inertia of the cross-section (I) is in 10. Which of the following, most nearly gives the vertical reaction at the roller, © in Newtons: A. 529 cS oa B. 460 B.7u7 U1. Which of the following, most nearly gives the moment reaction atthe first end, in Nevvion-meters ‘A. 396 cm B. 69 D220 12. Which ofthe following most nearly gives Uhe slope at the roller support in p.oo0ss1 D, 0.000386 ‘A, 0.000124 B 0.000268 are timber column with dimension 3 = 150 millimeters has eight of 340 meters The column is fed at Both ends (effective length fcr = 06). Th ims BOX sts grade Ang The proper of structural isber of Pelippine woods are given in Heute TSE12. The provisions of Uae 1992 Natal Structural Code of the PRlippines relevant {this problem ace gren in Fgare TNS. 13, Which the following get the clsafication of this colo iC Inermadiate ‘D, Notallowed by 1992.NSCP 14 Which he fllowing motes vs Hh llewsble oma sos, in MepaPascal: a tis B 776 15; Which ofthe following most nest gies the maximum compressive land, in klloNewton: ey <9 3 382 Bus Civil Engineering ‘Reference Vol. 2 271 Situation @ — A three-span continuous beam is shown in Figure ROS. The “sions of the 1992 National Structural Code of the Pi ievant previa foal Structural Code of the Philippines releeant TiloNewrtonsmeters! © Sone cms = DBs Situation 7 ~ An exterior column With service dead load = 760 kiloNewtons and. Service live load = 580 kloNewtons, and an interior colin wah serice onan tov fod = 90 eestor ae Fectangula footing whose outer end conn vend the oster face of theexteior cofuma, as shown se pes The ‘allowable bearing Pressure of the soil is 290 kiloPascals. The ©. 90 D, a7 following, most nearly gives the distance, in meters of the point of application of the resullant service load of the two columns, aeasured from the outside face of the exterior column: Whict of the folowing mast nearly grat the en |. Which af th follwing most neatly ger the misimum dimensions, in meters, of the combi ae = i i Tooting: Structure! Engineering 272 May 2003 ‘and Construction Ry = 0.053 meters t= 0.016 meters 555:meters $= 0.002077 meters? flowing most nearly gives the slendlemess ratio, above which with respect fo ateral buckling ‘compression flange, in MogaPasc ALS C85 B ots D. B31 24 Which of the following most nearly gives the maximum value of the ‘moment at the let suppor, in KiloNewton-meters: ‘ABE C205 B 198 D. 378 the beam is lon Situation 9 — For government projects, bidding is governed by. Presidential . Deere No Toa which state that for the purpous of detirniing the sverige of responsive bids bids higher than 100% ofthe AE or fore than Oe of he AE shall note considered The approved agency estimate (AAE) of the govemment project 280000, 000 The esas of he bicding sre a lows ea 25. Which of the following most nearly gives the maximum bid, in millions of ‘pesos, that can be consiciered a responsive bid: ‘A, 370 cus B. 336 235 26. Which of the fellowing mast nearly gives the minimum bid, in millions of pesos, that can be considered a responsive bid: ‘ABS C80 B. 302 D 27. Which of tile following gives the complet list ofthe responsive bids AL Bidders ACD © Bidders ABB B. Bidders BCE D, Bidders A.D, Situation 10 = The fol residential house, duration and cost ( are the activities for the construction of « nodes of the Network Diegram/CPM, time 5) for a normal and an accelerated program: © Activity 8 D. Activity J i ec for the project & PIE3700, which the following gives the ‘eduction in the number af days complete the proj Sune ne ie Bs BS FIGURES asian a Figure ME-88 274 May 2003 6euna=6m Figure ME-17 Figure TM-12 ‘The effective length L, shall be used! in design formutias given in this section. ‘Actual length shill be multiplied by the appropri efctve long ator to determine the effective length L. Allowable unit stresses of cross-sectional area of square or rectangular solid columns shall be determined by the following formulas, but such unit stresses, ‘hall not exceed the valuies for Compression parallel to the grain F. ‘Short columns (L./D of 11 or less) Poke Intermediate columns (L,/ D greater than 11 but less than K): ee] where K= 0.671 JE7F Fra, Long columas (L,/D of K or greater but less than 50} L/D shall not exceed 50, 30m Figure AC-21 Figure Re-6 ws #130 (ota) v= 2 SN (onfctored) | Section 5.8.33 of NSCP states that in leu of frame an: flloning ‘Spproximate moment ang shears are permitted for desian of beams nd one-way slabs (slabs reinforced to resist flexural stresses in only one Witection}, provided: 276 May 2003 and Construction eens ee ; (b)_ Spans are approximately caval, wih the larger of two adjacent spans not ‘Table 4-1: Working Stresses for Vis a ‘mar than 20 percent, Uneesoned tna) Tier of Pape Woot Teor GTS Oo "860 19 18.90 Negative moment at interior face of exterior ‘suppost for members bul integrally with supports, Where suppor ndrel beam... sme Wael 1 24) When support If @G01UMN ooo. onnnnnininemnne ny 1B. Shear inend members a face of first interior support... eee eraser mie) Shear at faee of all thar supports eens serine Malad 2 Where Li = clear span for positive moment or shear and average of aojacent clear spans for negative moment. 278 Way 2003 fang Construction bending moment about the strong axis Exratre belng an gee gle crete ot Figure ST-64 Section 45.145 of the 1992 NSCP gives the allowable stresses, in By = radius of gyration of a section comprising the compression ‘MegaPascals, for beams not restrained against lateral buckling a follows flange plus 1/3 of the compression web arta, taken about-eh 3 ans inthe plane of the web. 2. Tension: : d= depth of beam A=06F ‘ y= width of forge 1 thickness of flange 2. Compression: Th eget vate Sone By Foe 5-6) on (5 thd GEA) applicable but notmare dun oso5 FOR DOOC, SSO BONG, ee = END KC/RY? pe [2 ee le (45-60) 3 1055«10°C, L_, [aezp00c, When > 1,170,000) Be : (15-65) Ry Or, when the compression flange is rose-section and ie ara i il and. approximate ess han hat af the Pearce oe) BA wD HEL ee Bl MC RA Structural Engineering, ‘and Construction 280 May 2003 Solutions to May 2003 Examination 2 Situation 1 (1 to 3) eld mm E ‘nm 8 7 ‘ -M = Moment at B due to 15010 force M=150x ee M=18 kN-m Parts 2 and 3: ao ‘Omm oe 4am Ls an i fan in ~ — Voecnern I Moment at € due to 150 kN = Moment at due to Fy 180 i = Fa ai Fa=600KN Moment at 8 due to 150 EN = Moment at 8 das to Fe 150 fh = Fe x sis Fo = 450 kN Civit Engineering Reference Vol. 2 281 Dsitaation 2 at 6) 40M -01m TEMa =o ‘450(21) + 480(42) + 962:96(31.5) = 688.29(14.69) + (63) R=aN pee [EFr=0) (2Fa~ 9] ‘Av = 68829 N 17I9N (tension) (tension) triangle DCI (See the truss) nae wns, 978 oar = 180° -65° 47" = 68" 282 May 2003 EF =o) Fo: ina = 480 sin 65° Foye 469.19 N = 0 (1.5) + 480(21) 19 cos 47° (81.5) + 480(21) 200mm ‘Ay = 10020) = 2,000 mmz A= Ay + A= 8000+ 2,000 = 10,000 mm? [Ay = 22] 10.0007 = 8,000(20) « 2.000¢%0) 5-H mm 4=34-20=14 mm = 90-94 = 56 man ‘y= 10573388 mmt > moment OF neria Shearing stoss atthe neutral ax, V= 60,000 (given) 60,0000112,000) _ *" Sosraasqany 6 MP 284 May 2003, Situation 4 (10 to12) — Tisiteations (3 co 15) Past 1, Classification of column: 1 =3.6m=3600mm Tonk, L, = 0.673600) 1-= 2412 mm D= 150mm Kn os Jey E=731%10°MPa (from Figure TM-13) F.-956MPa (from Figue TM-13) pai K=0. K-06) K=1855 L242 D150 & = 1608 (greater than 11 but less than K) The column is an INTERMEDIATE COLUMN Port 2, Allowable Compressive Stress: Real (4) 91250 N_ > Vertical reaction at roller support [Ma = 5Re ~ 450) Ma=5(459) - 450 Ma =-280.5 Nem. Moment reaction at the first end: a = {Bana (given) = 0.000124 radian > Rotation at roller support Structural Eazincering Civil Engineering 286 May 2003 ‘and Constraction Reference Vol. 2 287 ee ___ GD Situation 6 (161018) Part 2, Value of a: Solve for X by taking mament of force about Pi: P= 760+ 380 = 1,340 KN Py = 1100+ 890= 1990 LN Po Prt Pe= 1340+ 1,990 = 3.330 kN X=3.317m a= X+0295 cc (0205 = 350m =k Put 1» Factored uniform load: w= Ld ty + 17ie = LA) + 17025) = 5.65 kim Part2 - Moment at C: Point C (negative moment) is at exterior face of first interior support with, ‘More than tere spans. Waly? eo t= Seu =6875m 2 c= SESGB 67m Part 3- Moment a Hz Point H (postive momen’ sat nd span with discontinuous end integral with support. y= ale 1, L,= 65 m (clear span for positive moment) 5.6565) 1 1 Situation 7 (19 10 21) REFER TO THE FIGURE NEXT PAGE Part 1, Effective soil bearing pressure: Mu = =17 kNem Structural Engineering ‘and Canstrucelan 288 May 2003 Part 3, Footing Dimension Such that the resulting pressure under the service load ip uniform, the resultant service load P must coincide with the centroid of the footing. ‘Thus, @=L/? 35i2=1/2 L= 708m ee Sars nt ‘Area of footing. Ay oe” Se yee 13559=7.084xW; W=191m ‘Therefore, the footing dimension is 2m x Tm Situation 8 (22 to24) hee ~ ooss # Co= 1.75 + 1.05(,/M) + 030(Mi/e? ‘My; =0.25M Mam Maj ite = (0.25M/ i) Mi/iMz=-025 (negative because they are of opposite sign) Co= 175 +1.05(-025) + 03060257 G=15005<25(0K) Civil Engineering ‘Reference Vol. 2 287 — art 2, Value of a: Solve for X by taking moment of force about Pi 3,350 X = 13400) + 1,990(85) X=3317m Fe=1008 | hima | fn | eee 288 May 2003 Sas) Par, Footing Dimension ‘Such that the resulting pressure under the service load is uniform, the resultant service load P must coincide with the centroid ofthe footing, “Thus, eee Urey Area of footing, Ay: 7, 5S 13.559 Lx 13.559 =7.084xW; We=19im Therefore, the footing dimension is 2m x Tm GO Situation 8 (220 24) (0.25) + 030(-0.257 y= 1.50625 <23 (OK) FrasmOOC, _, [FOS.OOOTT SOEs) _ i 28 FSR DOUG, _ [SSDOAIOTTSOES) _ 4. 5 738 289 L_ ,” f520,000C, F, a rectangular in cross:section andl its area is not less than fenaon ange, Fst larger vale (4560 anal (45-7) bat not OG and the compression flange is solid and. = RAT OL0G 0625) «5 sary 5} 83,000C _ 83,000(1.50625) = BEC, SALES sary iy C21 (O.015) 5 ‘Therefore, F) =98.5 MPa Part 3, maximum moment at left support: Allowable beading stress: In tension: Fy= 06, = 1488 0.002077 «10007 ‘= 208 584,500 N-mm = 208.58 kN-m 290 May 2003 Past 1: - ‘Maximum bid that can be considered! a responsive bid is P3%6,000)000 Past 2 ‘Minimum bid that can be considered responsive bid is 68,000,000, Part 3: ‘The responsive bidders are those with bid greater ‘than P168,000 000and lower than °336,000,000. ‘Thus, the responsive bidders are A, D, and E 1D Situation 10 (28 to 30) Nodes Past 1: From the network diagram shown, the critical path is path A-B-F-H- (Ca) ofthe project is Cue 22000 + 8,000 9,600 + 6,000 + 10,000 + 10,000 ++ 4,000 + 2400 + 4,800 + 15,000 + 2.000 ctvity to convert to accelerated program is the With the least cost per day te sccelevate O00 Peon per di +3 Ean =22,000 ae oe ri a00 pe ay T5000-10.000 «9569 per day = 4,000)/ (5 = 4) = P8900 per cay =P1.a00 per da a=e per day Thu cost effective activity to accelerate is Activity A. Part & Fora cost of P3,90000 ‘Accelerate A by = P B00 Accelerate B by = P 220000 Accelerate H by = Poon Total = P 3,900.00 ‘Thus, the reduction in the number of days is 1+2+1=4 days ‘achematics, Surveying, and ‘Transportation Engineceing ‘Seat Now 292 November 2003, CIVIL ENGINEER Licensure Examination MULTIPLE CHOICE 1. The magnitude ofthe vecor F=2i+ 6) Bhi: Aad © Bs B 36 : Doita nd off 108 288885 to four sgeifiant figures, BR aes enc B 1083 2. When the expression xt +a) 4 Set + br + 6 48 divided by (2 - 2), the When itis divided by (x + 1) the remainders 10. What is ‘constanta? Cr Bo D8 4 If versed sin theta =0,148, what s the value of theta? A. S157 cue B. S843" D. 1120 5. Whats the equivalent of 720° in the centesimal system? ‘A, 600 grads: C800 grads B. 750 grads D. 700 grads ‘spherical iriangle, angle B = 81° 50’ and angle C= 04 307. Ifsidee = isthe value of angle 4? © faieangle ABC a= 26. 7. Given the sides of a Ca = 36. . ‘opposite side ais 1027°. Compute the value of sie cin centintotres, 1525, C578 Di, 2045 8. Determine the area of a regular hexagon inscribed in. ‘of 170 square centimeters i D. The lateral ares of @ right circular cone is 386 square moters. If ts diameter is one-half ifs altitude, determine its altitude in raters Reference Vol. 2 293 C189 D. 225 area of a regular tetrahedron is 173.2 square centimeters. What id (4, 0}, the eurve is: be is 9404 units. Calculate BY? Bs ect JA. If the distance between points AQ, 5; 4) and Bfs, 3, 6) is AB = 5245 m, determine the value of x A cs Bs eT 15, Evaluate the following limit of (x ~4)/(22- v- 12) as xapproaches 4, veces 4 (2 -9)/(28- 4-12) 98 xapproaches 4. B. undefined D.1/6 16. The sum of two numbers is C. The produet of one by the cube of the other is to bea maximum. Determine one of the numbers, D.3ci7 ngular lot ABC has AB = $25 m, Jot is inscribed in tt such that the shorter side is on the 4.25 m ingle. Determine the maximum area ofthe rectangular Tot © 82em D.7. % 16m zi icle moves according to the parametii c the velocity ofthe particle when t= 2. © 14a2 D 1689 tions x = P and y = 20 B. Surface area generated by rotling theft quadrant portion ofthe oe. aS D, 6638 area bounded by the curve y = 6 cos x and the X-axis from x = Netox=a/2) AD B3 Bi 22. A nominal annual rate of interest of 7%, compounded continuously, has an theetive annual iets tale Sapien a A 25% B 7538 DB 74% ZB. What isthe difference between the sums of an annuity due and an ordinary anmutity forthe following data: 294 November 2003: ‘Transportation Engineering Periodic payment = PI00000 Terms = 30 years Payment intervals = year Interest ate = 12% compounded annus ee c P= (2) +(7 +8) =20198 Number = 108 288886 Rounded-aff to four signiti Rounded -off to five signifi 25. A horizontal platform, with radius of 25.m rotates about i center at a Constant angular speed of 10 radmin. The normal acceleration atthe ed Sepia x A063 my ©. oows B 0082 m/e Doni 2%. A horizontal force of 250 N pushes a a 22 m/s after travelling 120 ction between the block and th ‘A 028 ‘ © 021 B a D. 036 28, ‘The cross-sectional area of road with width of 10 m is 33.1 square meters, ‘The cross-sectional area is a8 follows: fdr arene susinsg BOERS spaces WO 616 +0) S200: 7 tO 12 Determine the value of Z A 32m 24m B. 28m D. 16m 28. closed triangular traverse asthe folowing data: Bearing Distance a Noor E 11000 b=2-(5)=7 Bc Due South Neo w Versed sin @= 1 cos 0= 0448 cs d= 1852; 6 = 81.57" 2.000 square meter is cutoff siting fom comer A to point ‘online EC. What is the length of line AF? A 87835 m C. 868.14 m B. 893.25 m D. 91475 m Note: Versed cas 0 =1- sin END“ ‘The units of angle in centesimal system is grads, where 90° = 100, meri system is gra gral. 0 = 500 grads az sind _ sine sin b= 09929; b= 83.175° Using Napier’s analogies: tan $A = 0537018; A = $6* 28° Giver: 4=363,b=23.9, 4 =1027" Solving for angle By sine lawn ' (ere rane See snB Gna) ‘snB ” Salar B= 3996" and dos” Since a plane triangle can’t have two angle more than 9, use B= 39-96" Angle C= 100° 1027" 38.96" =37.38 Solving for sey sine aw: ware "mie? em 2257 em ‘Area of circle = xr? =170 187.356 Avenegon™ V7 356) si 60" 6 Avceagon = 140.6 cen? Meni. _ $o* en - ia! 1 Venlo S67 cu = * ‘Reference Vol. 2 297 ‘Ow Lateral area, A, = xr. = 386 r=1287 = >a) hear 20) L 178 Lervi7 70) Substitute L of Ea, 10 Eg. (1} Eran a 122.87; = 5.459 Then, L= 545917 = 22.51 m on Surface area = a 3 = 173.2 2=1em Altitude, A= af = 10,J% 8:16 cm on ‘The curve Ax? + By? + F=0 isa conic section Dividing both sides by a: ses bys f= AL (A, 0} P+kOP+ hao f=46 >E4.( F= 0 is a conic with either be a circle, un Since the curve passes through (0, 3) and (4. 0), obviously, the curve must be an ellipse, as shown, ‘Mathematics, Surveying, and 298 November 2003, “Transportation Engineering, “The distance d between two points Pi(s. yu, 2). andl Pees yo) ist P+ Ga P+ 2 2+ B19 + 6-4)" (xu yas 2x) and Pas, ys 23) i: 3999-4 Sa sans 1/7 3985 — 3999-12 Letx and y be the numbers rey =C of ymax Product, P= 33 Pax\C- x)= Co—x1 #=sca-ar~0 #QC-4)=0; | 38= 0; x= Ofabvard) 3C-4x=0; x= 3/4 Another Solution: For maximum product of x xy where x+y C: € € aan men where n= Land m= 3: c x= 753-308 ‘Mathematics, Surveying, and 300 Nowember 2003 ‘Transportation Enginecsing as nt 2 r=4(1-sin®) — cardiod es » S= [> =2) ffe-snme 1 ao § = * 4 hs cos (90°) ‘Vertex at (0,2) open down «(Gs yar-get ay a dS= fix (ha) de as fieqe dasa dee fia ge ax acts [ indent 209 [ev eke y a0" ] ao}x anise 2 |(egs*) jun Sin] 20027 ais A= 6(ein 90° sin 30°) Aw Saq.units ER=er-1= 8-1-0725 = 7.25% = 100,000f(1 +0.12)""= 1] O12 a+) = Lonooo(d +0125" —1) aut +012) Difference = P8,069873.55:- F7,205,244 24 = P864,629.91 Or 2s FC = PA20,000; n= 6, $= P5000, m= 1 By straight-line method: Dye d= FO=SV _ 420000 ~50,000 SOR = POL.556 67 302 November 2003 Mathemattes, Surveying, and “Transportation Enginecring mas 26 By sinking fund method: (FC-SV)i | (420,000 -50,000)(012) © Gyan (ronan a ay Dyad= PAS 59851 By SOYD method: oven aca sum= Barn) =2a+6)=2. Dye (emono soo AAGI=F =p 157143 (0K) ‘Thus, the SOYD method was used. N=W-Psins N=u7is- aDsins* N= 1165080 For inypending motion fEhu=9) u(116 208) = 40 cos 50° = 0.2807 pees {= 10 admin x 1 min/@0 c= 1/6rad/oac ae (1/67 25) = 00698 myst w= Koon m2 Work-Energy equation from A to B: RE4+ Work: Wh = KEx + is 084m (+ 250(120)- 600 (120) + 0= ve 22)2 w= 021 AxAt Art Art Ay BBE = SANRA + HEKA + ONC) + HONE. Acce = ¥1000)(a) sin 60° = 280,000 d= 64663 In triangle ACE: 22 = 10002 + 646 63° - 2(1000)(646.63) cos 60” = 87835 304 November 2003 Geocechnical Engineering Seat Nos - ‘alarated clay lier Bes water Content ol ; ‘7%, The specie gravity of the sold parices 28h, ont OF CIVILENGINEER Licensure Examination 7. Determine te dey ofthe clay int) Sanday, November 23,2003 (2:00 p.m. 06:00 pm, ae a iss D153 “ Determine the total vertical strss atthe bottom ofthe clay lay, in KPa. = c SETA "As 387 5 ot D396 JRSTRUCTION: seit ts comet newer a ac of he lowing guetinn 9, Determine the effective vertical stress at the boty of the ely ayer, in kPa, Mar {or each tem by shading, the box corresponding tothe letter ‘a 200 crus ‘of your choice on the answer sheet provided. 8 185 D. 138 STRICTLY NO ERASURES ALLOWED. Use peniino. 2 only : Situation 4 A fully saturated clay sample hus a mass of 1525 grams. After over ‘SITUATIONAL : : lying i subs war vedared Wo LOS) grams. The pele ave sll ices i627 Situation 4 = A. borehole log profile in a construction project is shown in Fi 40, Ehlcalate the natural water content of te sample in percent. SME, The onstruction wil imprt net ses of 12 Neat ‘A. 52 cos. Use unit weight of water equal to981 KN ce Dao 4. Determine the nearest value to the Buoyant unit weight of the clay in Ti. Calealato the void mation percent N/a a 136 ere) ©. 957 Bits Daa Br D. 1012 12. Calculate the porosity in percent 2. Determine the nearest value to the effective vertical stress, in KPa, at the A cs tnid-height of the clay layer. B36 Be ‘A. le c sa 2) Went a ie aceon Stan one nepal sonsslidaled clay Lager (GMMR cepaced tS Shea’ Ge vols womcdie ee ee Oe ee isthe average x slay layer ‘quired to caso the soils according to ied Soi Classification fentimeters, Compression index C, = 0.009(LL - 10) nlsis. Use the atlached USCSS, aT cosa 6 the value ofthe coefficient of uniformity of sol A a Ba Da ‘A 3a C32 cing esionlgs soil a hich moe Situation 2 ~ A retaining wall 8 m rts cohesionloss soil having a 14. Which of the following classifies soil A? ei of 160 g/m ag ibang retemce oS and veld fata © ACP ae CSM GG. The surface the sll is honzontl and level with the top ofthe wall 8 SP Ba Neglect wall fiction andl use Rankine's formula for active prossire of & 15. Which ofthe following classifies sil 8? cobs ASP cop 4 the nearest value to the total earth thrust on the wall in KN por ee iM he soil is dry. ‘Detrmine the nearest val to the thrust on the wall KN per Hl meter fowing to inadequate drainage, itis waterlogged to 4 level 5m below the surface 6. Determine the nearest value to the hetght above the base ofthe wall where the thrust acts during the waterlogged condition 35m C 175m 306 November 2003 Geoestainstnnpeeesiog Sener Reference Vol. 2 307 18, What is the total amount of water that percolated after 55 minutes, es Grain Size Curve ‘A. 186 co Sle 7 B86 ce D: 26.c¢ = ean sees Bee 1, 2 and 3 are Has ee in parallel, oa charge ol pes is equal 061 m?/ece, compu win 5 Se ee aa ee = a” Pipelines ‘Length (n) Diameter (mm) ee ra mth etn (ns) 5 2 30 200 Hey 3 _ 730. 19, Compute the ite of flow in pipeline, in L/s. he aio Ee aw B40 D. 135 Be 2. Come ern offing 2a A361 o BMD 21. Compute the rate of flow in pipeline 8, nye AL 40 ® B. 65 a ls Particle Diameter, mm prs cm projecting above the 22 Derm teheightof de 6 A testis setup ar shown in SM-23. A cylindrical mold £" in diameter ed with sik to height Hy = 02 ft, whose coefficient of permeability ky = 3.6 10+fi/min, 23. Determine the specific gravity of the wood. ‘A. 056 C, 043 ‘A. second coaxial mold is placed on top of the first mold whose inside D. 052 the weight ef the wood if its cross-sectional area is 20 can x 20 © WIN DL 321N Th test stp ise permeamea of santa ead, West ‘maintained at s level f= 1 sidered that the systern cer and cocficent of permenbili Situation 9— Water flows at thw rate of 12 1n?/s in an almost level channel that is ‘m. The depth gradually increases from 1.0 40 1.10 m over a length of flow of Sm 25, Caleulate tho head lost in meters. ‘A, 010523, Coons B. 0945 D, 0026s 26, Calculate the slepe of the energy grade line ‘A 0083 sere 0083 B. 000s D, 00018 2, Calculate the value of the roughness coefficient ‘A 000095 C. 00135 D. ogra B, 6.023 » 104 D 828-10! Pree Hydrauics and ‘civil Engineering 308 November 2003 Gcotechnical Engineering: figtccence Vol. 2. 309 Situation 10 A wooden vat b in the shape of a frustum af « cone, 2m diamete the top and 41m diameter atthe bottom, and 3m high. Iris provided {iva steel hoops one atthe top and the ather atthe botiom, Ifthe vat is with oll having specifi gravity of 0.20: 28, Besemaine the yoni fore onthe the va 8bm p= 176 300 28. Determine how high i fhe said force above the botiom ofthe vat? ‘A 11S © 070m 106m 085m D. 090m 30. Calculate the force in the bottom hoop. | ym 1d Figure 5696 the unit weight of glycerine in kN/m. © iss D Lar 99. What nthe specitc gravity of glycerine? AL 132 eee C.11z 2 ie D136 Situation 12 - A d-mediameter open cylindrical tank, 6 m high, is filled with water. | The tank is tilted to a pasition such that its water surface will cut the diameter of the base. 34. Determine the volume of water left in the tank, in cubic meter. £8 © 16 4 DIB 35 Deere the depth of water in the tank when it is restored to its upright “* 127m 6. White the tank is fall of wate anxis such that the depth of wat Figure SM-23, and. Civil 310 November 2003 csoumsestzosincetot | Referense VoL2 312 ——— UNIFIED CLASSIFICATION SYSTEM (USCS-54) Solutions to November 2003 Examination lc Copsnai4 Gopsrpanoasmates ale af reer bh oi Heal 2 i eee ee te) BA] = eae ene? eee ii Ae FHL | ereees ormronnummn eal. Bees ji ae aaa Brae £8 fils ai [eesesc [ero at cH 2. = ale Rommemexeen | [pee , For Normally consoliated elay: eheeree 5 MS | serriee nes * Ll = 0.002145 - 10) Ly) eee | - = "= effective vertical stress at mid-height of clay ay ul: Beer | * al Posi vera esl ght of ay ya ia] |e ee | i : 2 Eee tH 1. o i 8 jlo ee spare Aree) Sree tl : are Dsitwation 2 (to 6) ae | rr ae ‘aa nad sentation, Ge ASTM Deegan Part k: waren som 5.696)(35) = 16.195 kPa = 28.34 KN; yin d+ 35/3 =5667 m yee M45) =2.25m, yn45/3-15m Fem Vine HE = 3A yr d5/3=15m Total thrust, Fe A+ +1 + Fe ‘Total thrust, Fr= 28.34 + 7288 + 29,421 + 99.33 279.97 KN Location: Frx 9 =2Fy ‘D997 = 28.345 467) + 7288(2.25) + 29.021(1.5) + 99.33(1.9) J =2251m Giv os7 5= 1 (saturated) G=2e k= 20m [GMC=Se] 2.84057) =100) e= 16183 _ 284+1(1,6188) ter Ty 618s Yo = 16.7026 IN ) trated unit weight) G-27 M=1525 grams M,= 1053 grams Pat: ice Mus. 1.526=1,055 @ Situation 5 (19 to 15) Soil A: Dy=010; Dig 017; Du=052 313 Mydreutics and 314 November 2003 Passing No, Asieve = 100% (mone than 0%) ‘Sans ‘The soils either SW, SP, SM, or SC .-32<6 — C.=0.908 (not between and 3) Since the soil does not set both criteria for SW, the soils SP Grain Size Curve sas 2 Percent Passing, % 8 a 6 8 Soil B: Uniformity coefficion 2 2 Coefficient of curvature, C.=

6 C= 138 (between 1 and 3) ‘Thorefore, the sofl is SW Situation 6 (16 w 18) ee Sand, k= 2.7 «10? Wn St, k= 36 x 107 rYmi| Part: Path a Qakia Ke 3.6 10+ ft/min t= h/L=125/05=25 A= $2=3 0127 fe Q=321044 1015 ~ 10 £8/min 7.5% 104 R/min, 0.075 40 Hy= 6.9518 x 10+ Emin Past 3: Y= Qx t= 1.5167 x 104 f0/min x 55 min 1¥ = 0.00834185 f° x (100 cm/3.25.) V=236aem' 2 Situation 7 49 t0 21) Q+Q+Q=061 > Bq. (1) = py 20S 2OUAEBO}A* __010826(7)(600)02," Was Wi aoe 015% Q:=225510, > Ea. (2) j QOS ANTES? = 20826(/)(600)0,* 010° 015° = 03860; > Eq.) InFq.(1); Q,+2.295101 + 092860) = Q, = 0.1685 m/s = 168, InEg.(@): Q:=2.2951(1685) = 386.7 InEq.(@): 1 =05246(108.5) = 58.695 Geotechnical Engineering Civil Engineering Reference Vol. 2 situations 22 Draft= ht In water: eg onto 1 Hawt @h=5 In glycerine: 75=jfoed 135 135h- 10.125 > Eq. (2) Dison Sana) 10 + 0.6585}((0.2)(02)(0.1469)] Weight = 37.8 Situation 9 (25 to 27) Mydroutics and Geotechnical Eaginecring Qh= (75x L0NRSH.07S) = 140625 10466) min = Qs + Q) = 1.1085 x 105 + 1.40625 x 104 515167 <104 AYmin 316 Nowember 2003, ky 69518 «104 fyfmin Part 3: Ve Qx t= 1.5167 x 104 f8/min x $5 min Vom G.0084185 fe (100 emn/'3.28 fs Vom 2364 cm {Situation 7(19 to 21) e 8 oe eer 8 ey + QQ) = 0.61 > Eq.) ih Aa) O.0R26(f)(480)5" __9.0876()(600))? 020° O45? Qr=229510, > Eq.) O86 /)PSOV2.™ _ 0.0825(/)(600)0,7 ‘010° 015° Qs~032460; Eq.) (i= i) InEg (1); Qi+229510, + 032460, = 061 r= 011685 me/s= 1685 literyfese In Eq, (2) Qa 2:2951(168.5)~ 386.7 liters/sec InFq. (3); Qa= 0:3246(166.5) = 54.695 liters/sec Civil Engineering : voL.2 ‘DDSituation 6 (22 to 24) pans heehee ore sp. ge of liquid In water: h-Sah Saw 1 Seat == 5 35i- 10.125 > Eq. 2) Bi seat Sal) F h-S = 1354-10125 ha 16hem| Weight = Yet Vast Weight = (6610 « 0.6585)((0.2\(0290.1468)] Weight= 37860 “Situation 9 (25 10 27) Hydraulics and ‘Geotechnical Engineering oo 4m/s 2 = 08155 m 2% =e 22 0678 25 $= 08083 > Slope of ECL lead! lost, HE. = SL = 0.00836) = (0415 mm. Value of nen Lanse y= DAME = At 96368 5 5189 m/s 2 2 anna= Losi oom w= aos Reference Vol. 2 GBituation 10251030) Part 2: Fx aod 320 November 2003 ceorecnnica! Eneinecring © Situation 11 (31 to 33) Pent ‘Weight, W= Mase W= 1500 x 981 = 14,715 N W16715kN Past 2: Ww uss Unit weighty= “Y= ‘Unit weight y= >= <5 Unit woight, y= 12.366 kN/mm? Part: Specific gravity, s= 2£ Specific pravity, 2. Situstion 12 (3410 36) eames! Civil Engincering ‘Reference Vol. 2 321 Part: ‘When the vortex touches the botiom of the tank, the upper sutface will touch the top rim of the tank. This is because the volume of paraboloid is ‘one-half the volume of circumscribing cylinder. amo = 518 rpm ‘Civil Engineering Reference Vol. 2 325 R= 005m [00am 2. Using the appronimation that the radius of gyration 7 i appro frase = 0.002077 me qual one oth the wich ofthe fargo which oh fllaing may Be 16. Which of the following most nearly gives the slendemess ratio, below which foneae the bam s short wih espect to lateral ecling B. 200 mm DB. 20nm ‘a 38 ‘ 2A. Using the protiminary value of the flange width sbuained B 753 D7 ‘the following most neutly gives the requeed plate thickness 17. Which of the following most nearly gives the allowable flomural stress in the Fete ge eee tee ee Compression flange seording wo equnon 45-6 or yuan 45-6, on a ‘pphcable. cere A999 MPa © 66.7 MPa quantities isto he prepared for B e63MPa D,754MPa Unit weight ol seals 7800 ka/ah i 1B. Which Of the following most neary gives the aximum value of the over for reinforcements 100 mm. i ieee momentaat the left support, in kiloNewtan-meters; ‘ 25. Which of the following most nearly gives the iy sone ‘A 2547 C3021 uamaerreinorcing ba eg/an 7 O° Se Seat weleht of Ge 16mm 2788 D574 : BS Gla < - LIS Situation 7 - A simply supported rectangular reinforced concrete bedi has a span. ich o ong see Sf gan, width of 220 mam: elects clepih ef S00 mon and toa depth of 2D %: Which of the following most nearly gives the quantity of 16mm-diameter tn The beam ip enierced wah berm clamoto bars” concrs ‘A 159 34 firength f-~ 28 MPa and the steel yicld strength j= 415 Ma. In addition 1 345 Douay te bg kis and its own weight the boar entries'a 150smmesthick concrete 27. Which of the following most nearly gives the quantity of 16mm thbibet Sm wide The unt weigh ofconcees24}N/ av, The bea bars ink earner ane to beanalyzed and designe! using the Strength Design meth . 19. Which of the following most nearly gives the total ervice (anfactored) as rare uniform dead load acting onthe beam, fn KN/m Aas Cs Situation 10 ~ Government construction contracts 8 152 D223 cee tution fontracts are governed by Presidential 2, Which ofthe following most nly ges he nominal moment capac meal rovided that nd major pay’ tem shall be ee She bem naNn as ty more Ered cst ay define! as pay items a c= representing, at leat 20° of the AAE' or the fist two steme hevlog Ie 4 highest percentage of the AAE or those indicated or specified the 21. Which of the following mest newly gives the maximum uniform service Instruction to Bilders, For this problem, the AAE of {unfactore) live lod acting over the entre span that can be supported by si iSaulken certain government, te he ae %. Which ofthe following most nearly gives the amount at which an ten 8 2a7 D788 9 is. welded built-up 'H-shape 13 Situation 8 ~ A tension member in a bridge ‘long and 400 mm deep. The member is composed af three plates welded) together. The maximum load on the membe Kk tension and the minimum load is 250 KN compression. For this problem, it can be assumed that compression is not critical. The member should be designed for a design ood equal to the maximum load plus one-half the mincmum howd. The llonable trite sess 85 MPa. ‘The maximem slendemess ro IR 0, (L/n) 200, D. on gj moet nealy gives the maximum increase in quantity ee is unchanged? 22, Which ofthe following most nearly gives the minimum area of the member tees A. 8756 mm? . 9867 mm? Co B, 10786 mmt D. 9351 mm? ne

Вам также может понравиться